GI/Nutrition Qbank

¡Supera tus tareas y exámenes ahora con Quizwiz!

A 54-year-old man has cirrhosis, with obstruction of the portal circulation within the liver. Portal blood could still be conveyed to the caval system via which of the following?

Azygos and hemiazygos veins. The esophageal venous plexus, which drains into the azygos and hemiazygos veins within the thorax, has anastomoses with branches of the left gastric vein. Following blockage of the portal vein, portal blood thus may enter the superior vena cava via the azygos system. Other important portacaval connections include the superior rectal vein with the middle and inferior rectal veins; paraumbilical veins with epigastric veins (engorgement of these vessels results in caput medusae); and the colic and splenic veins with renal veins and veins of the posterior body wall. Cirrhosis occurs as a result of blockage of the hepatic venous outflow. This results in expansion of the splenic and gastric blood volume and expands the hepatic lymph formation. This leads to intrahepatic hypertension, which results in the increase of sodium and loss of effective blood volume. There is also a decrease in the production of albumin, which also complicates the picture of ascites that occurs as a result of this portal hypertension. If there is blockage to the circulation via the portal system, alternative drainage of the liver can occur through the azygous and hemiazygous veins.

A 32-year-old man has a 15-year history of celiac sprue. He admits to being noncompliant with the diet prescribed over the past 6 months and has lost 15 pounds during that time in association with frequent diarrhea. A microcytic, hypochromic pattern is observed on a peripheral blood smear. Serum ferritin value is 15 ug/L (normal 20-250 ug/L). Which of the following is most likely to occur in this patient?

Celiac sprue will produce signs and symptoms of malabsorption of the proximal small bowel. Iron (along with folate and calcium) is preferentially absorbed in the proximal small bowel and is not well compensated for in the distal small intestine when there is proximal small bowel malabsorption. Vitamin A, D, and K deficiencies will occur but do not produce the described symptoms. Iron-deficiency anemia is characterized by a microcytic, hypochromic pattern on the peripheral blood smear. In addition, the serum ferritin level of less than 16 ug/L is strongly suggestive of a microcytic process. It should be remembered that the red blood cells become smaller and the ferritin levels fall before the fall in the hemoglobin and hematocrit.

A 45-year-old chronic alcoholic presented to the emergency department 5 years ago with 24 hours of epigastric pain radiating to his back, nausea, and vomiting. He gradually recovered from that acute episode. Over the next 5 years, he is repeatedly admitted for similar symptoms. He then presents with gradual onset of weight loss, midabdominal pain radiating to his back, and steatorrhea. Which of the following conditions has most likely occurred?

Exocrine insufficiency of the pancreas. Severe epigastric pain radiating to the back and accompanied by nausea and vomiting suggests pancreatitis. Pancreatitis is most likely to be encountered in alcoholics (such as this man) and patients who have biliary tract disease. This patient has a history of recurrent alcoholic pancreatitis. The development of gradual weight loss, chronic pain radiating to the back, and steatorrhea suggests that he has now developed chronic pancreatitis. This condition is often complicated by both endocrine and exocrine insufficiency of the pancreas, secondary to loss of much of the tissue of the pancreas to the disease process.

A 36-year-old schoolteacher comes to the clinic for a new-patient evaluation. She teaches second grade at a local school and just recently moved to the area. She has no current health complaints but reports a history of hepatitis. She apparently contracted "chronic hepatitis" from a blood transfusion received shortly after the birth of her first child 20 years ago, although she is unsure of the details of the disease. She was told she was infected, but she has always felt well and she doubts the diagnosis. Since then she has not suffered any symptomatic liver disease, and her liver function tests have always been normal. Review of symptoms fails to reveal any evidence of liver disease, and physical examination is unremarkable. A review of medical records that she has brought with her reveals the following: Hep A ab neg, HBsAg Neg, Anti HBs Pos, Hep C ab Pos. In addition to disease education and counseling, which of the following is an appropriate intervention for this patient at this time?

Hep A vaccination. This patient is extremely likely to have chronic hepatitis C but is currently asymptomatic. Though asymptomatic patients without any elevation of liver enzymes do not require treatment, they do need protection against other forms of hepatitis; that is because, if infected, they are at a much higher risk for fulminant liver disease.

A previously healthy 11-year-old boy is brought to the health care provider with a fever and persistent vomiting for 4-5 days. Initially the emesis was clear, but now it contains streaks of bright red blood. Physical examination, complete blood count, and serum electrolytes are all normal. Which of the following is the most likely diagnosis?

Mallory-Weiss tears occur in the lower esophagus after forceful or protracted vomiting and involve only the superficial mucosa in the lower esophagus. In most cases, bleeding is painless and resolves once the insulting incident (i.e., vomiting) resolves. Mallory-Weiss syndrome is a tear in the mucosa of the gastroesophageal junction. It occurs as a result of a sudden rise in abdominal pressure that results in the tear and subsequent GI bleeding. This syndrome occurs following forceful or long-term retching, vomiting, coughing, or straining. Treatment is supportive, because most cases are self-limited.

A 63-year-old man in apparent good health is convinced by magazine articles and TV programs that he will live longer if he takes one aspirin tablet every day. After about 3 weeks of doing so (325 mg/day), he begins to notice bright red blood on the toilet paper when he wipes after a bowel movement. This does not occur every time, but only when, for whatever reason, he has to strain more than usual. He has never had any discomfort referable to hemorrhoids or any bowel pathology that he is aware of. Anoscopy and digital rectal examination show external and internal hemorrhoids, none of which are bleeding at the time of the examination. Which of the following is the most appropriate next step in management?

Normal colonic mucosa does not bleed in response to anticoagulation, but lesions with a predisposition to do so may be more likely to become symptomatic at such time. The bleeding lesion in this case may indeed be the internal hemorrhoids that were diagnosed, but a malignant source has to be ruled out before making such assumption. At the very least, the patient needs a 60-cm examination with a flexible endoscope. If one of the answer choices was colonoscopy, that would be the next best step, because this allows visualization of the entire colon. Because the patient is older than age 50 years, a colonoscopy is warranted as part of the evaluation for potential colon cancer.

A 70-year-old man complains of chronic heartburn. It is painful for him to bend over, and he sleeps on a wedge-shaped pillow to try to reduce the burning sensation. Which of the following agents would be most efficacious in reducing his symptoms?

Omeprazole (Prilosec) is an irreversible inhibitor of H+/K+/ATPase of the parietal cell (also known as the proton pump). It is used in the treatment of peptic ulcer disease, reflux esophagitis, and Zollinger-Ellison syndrome. Omeprazole can reduce daily gastric acid secretion by more than 95%. This drug is of particular value to patients who do not respond sufficiently to H2 antagonists. Because of the profound reduction in gastric acid secretion, patients can develop modest hypergastrinemia. It is the most effective way to treat GERD burning, because it maximally suppresses gastric acid production.

A 34-year-old tax lawyer comes to the clinic complaining of difficulty swallowing. On several occasions over the past few months, he has been aware of meat becoming stuck in the mid-chest immediately after eating. After each episode, he has several hours of chest pain that resolves gradually. On 2 occasions he induced vomiting to obtain relief. Over the past 10 days the difficulty swallowing has become worse, and he now has trouble with even soft foods. He has been taking ranitidine (Zantac), magnesium hydroxide, and omeprazole (Prilosec) for 4 years but has remained symptomatic despite these measures. He has smoked 1 pack of cigarettes daily for 15 years and denies any alcohol use. Physical examination is normal. Which of the following is the most likely cause for these symptoms?

Peptic esophageal strictures. This patient presents with symptoms consistent with gradual luminal narrowing of the esophagus after many years of gastroesophageal reflux disease (GERD). These symptoms suggest the development of a benign peptic stricture. This may occur even if the patient is on medical therapy to reduce acid secretion, because many patients will continue to produce acid despite standard medical regimens and may require very high doses of proton pump inhibitors. Treatment consists of endoscopic dilation of this stricture and continued aggressive anti-reflux therapy. Long-standing, chronic GERD predisposes to development of peptic esophageal strictures, even during medical therapy with proton pump inhibitors. Carcinoma is less likely in young patients with no history of constitutional symptoms or weight loss.

Ten days after having an exploratory laparotomy for blunt abdominal trauma, a 53-year-old man develops daily fever spikes to 38.9ºC (102.0ºF) and 39.5ºC (103.1ºF), as well as leukocytosis. At the time of surgery he was found to have a seat-belt injury to the small bowel and required resection and re-anastomosis of the affected area. He then had an uneventful postoperative course until the fever became apparent on day 10 of postop. He has been tolerating oral intake since day 5 of postop, and at this time his abdominal examination is unremarkable except for the fresh midline wound. Rectal examination is also unremarkable. The abnormality responsible for the fever will most likely be demonstrated by which of the following studies?

Repeat Blood Cultures. Fever that starts about 10 days after a contaminated abdominal surgical procedure is most likely caused by a deep abscess, either pelvic or subphrenic. The former has been ruled less likely by the patient having a normal rectal examination; the latter can best be demonstrated by CT scan.

A 9-year-old boy is brought to the emergency department by his grandmother because he suddenly developed a fever with shallow breathing, vomiting, and mental confusion. She is beside herself because the child has come to visit over the holidays and had been perfectly well before today. He has never had any problems with his health except for a skin condition present from birth that she describes as "alligator skin." When you ask if she gave her grandson any unusual medication or remedy, she admits to having applied a homemade paste to his skin that was supposed to get rid of the heavy scale. She states that there were several affected family members in previous generations and this remedy has been used to treat them without ever having caused problems. She also insists that it could not have been the cause of any illness, because it was really just a bunch of aspirin tablets crushed and mixed with petrolatum. The boy's temperature is 39ºC (102.2ºF), pulse 120/min, and respirations 35/min. He has vomited twice while in the emergency department and appears to be dehydrated and lethargic. Blood glucose is 45 mg/dL and serum pH is 6.9. Which of the following is the most appropriate next step?

Salicylates have analgesic and antipyretic properties when used systemically, and keratolytic properties when used topically. This patient has developed salicylate intoxication secondary to absorption of large amounts of salicylate through the skin. Many skin conditions that are characterized by hyperkeratosis are treated with topical preparations that contain a certain amount of salicylic acid (ranging from 1 to 40%, depending on the indication). They should never be used in children, however, because the surface-to-volume ratio is very large and even a relatively small amount absorbed through a large surface area will result in significant systemic levels. Even in adults, preparations with a significant amount of salicylic acid should be used cautiously, and either for a very limited time or only on certain body surface areas at a time. The clinical presentation and physical findings will vary with the dose of salicylates ingested. Vomiting, hyperpnea, fever, lethargy, and mental confusion are seen in mild salicylate poisoning. Convulsions, coma, and respiratory and cardiovascular collapse are seen in severe salicylate poisoning. Hyperventilation, dehydration, bleeding disorders, seizures, and coma are seen in chronic salicylate ingestion. Arterial blood gas analysis will show metabolic acidosis with respiratory compensation in children and a respiratory alkalosis alone in adolescents. To enhance the excretion of salicylate, the intravenous route should be used to administer bicarbonate and raise the urine pH to 7.0-7.5. If the salicylate level is greater than 100 mg/dL, hemodialysis may be indicated.

A 61-year-old man presents with colicky abdominal pain and vomiting of 3 days duration. On physical examination there is moderate distention, high-pitched hyperactive bowel sounds, and a 5-cm tender groin mass. On direct questioning he explains that he has "had that bulge for many years." He has always been able to "push it back in" when he lies down but for the past 3 days he has been unable to do so. His temperature is 38.9ºC (102ºF) and white blood cell count 12,500/mm3 (normal 5,000-10,000/mm3). Which of the following is the most appropriate management at this time?

The clinical picture is that of a strangulated inguinal hernia. If he only had the tender mass without signs of intestinal obstruction, he might have omentum trapped. If he had no fever with a normal white blood cell count but a tender mass, he could be obstructed but without strangulation. The combination that he has, however, is clearly that of obstruction with strangulation. He needs urgent surgery.

A 56-year-old man has been having bloody bowel movements on and off for the past several weeks. He reports that the blood is bright red, coats the outside of the stools, and appears in the toilet bowl even before he wipes himself. There is also blood on the toilet paper after he wipes. After further questioning it is ascertained that he has been constipated for the past 2 months and that the caliber of the stools has changed. They are now very thin compared to the usual diameter of an inch or so that was customary for him. He has some minor discomfort. Which of the following is the most likely diagnosis?

The combination of red blood coating the stools and a change in bowel habit and stool caliber spells out cancer of the rectum in someone in this age group. The following are the combination of symptoms in the presentation of rectal cancer: hematochezia (bright red blood in the stools) and/or enterorrhagia (intestinal bleeding), a change in bowel habit (constipation), and a change in stool caliber (pencil-like feces).

A 9-month-old boy is brought to the health care provider's office because his weight is persistently below the tenth percentile. His mother states that the infant seems to be hungry all the time and usually consumes 8-12 oz of formula every 2-3 hours in addition to some table food. He also has frequent, bulky, and malodorous stools. He does not take any medications. A malabsorption syndrome is suspected. The results of which of the following tests will most likely be abnormal?

The most useful test for the 9-month-old infant in this clinical vignette is fecal fat quantification. Fat malabsorption is likely, with poor weight gain and frequent malodorous stools. A detailed family history and physical examination are also useful in evaluation of fat malabsorption syndrome. In a stool sample collected over 3-4 days, fat excretion should not exceed 15% of dietary fat in an infant, or 10% in an older child. When a malabsorption syndrome is confirmed, a sweat chloride test is indicated to evaluate for the possibility of cystic fibrosis, which is a common cause of fat malabsorption.

A 62-year-old man has had gastroesophageal reflux disease diagnosed by pH monitoring, which has been present for several years. He has been less than totally compliant with medical management, which he follows only when the pain is bad but discontinues when he feels better. Endoscopy and biopsies show severe peptic esophagitis, with Barrett's esophagus and early dysplastic changes, but no overt carcinoma. Additional tests show good esophageal motility, with low pressure in the lower esophageal sphincter and normal gastric emptying. Which of the following is the most appropriate treatment at this time?

This man has indications for surgical intervention, but all he needs is an antireflux operation. By far the most commonly used procedure is a laparoscopic Nissen fundoplication. This procedure is performed laparoscopically. GERD that does not respond to conventional therapy may need antireflux intervention. The procedure of choice is laparoscopically performed Nissen fundoplication in which a portion of the stomach is wrapped around the lower esophageal sphincter area to reinforce this area and prevent reflux. Preventing reflux will allow healing of the lower esophageal mucosa with the hope that any dysplasia will fail to progress.

A 22-year-old sociologist returns from 6 months in Jamaica and notes a 6-pound weight loss during her first month back home. She reports that her appetite is normal. She has developed symptoms of mild abdominal cramping and bloating after meals and frequent greasy bowel movements. Physical examination is consistent with evidence of recent weight loss and abdominal examination is normal. A stool specimen is guaiac-negative. There are scattered ecchymoses on all 4 extremities. Laboratory results reveal an albumin of 2.9 g/dL (normal >3.8 g/dL), INR 1.9 (normal 1.0 in patients who are not taking warfarin), and normal liver function tests. A trial of a gluten-free diet is attempted. There is no change in symptoms over the subsequent 3 weeks, and she has an additional 4-pound weight loss. Stool cultures for enteric pathogens and ova and parasites are negative on 3 occasions. Which of the following is the most likely diagnosis?

Tropical sprue causes progressive villus atrophy in the small intestine that is similar to celiac sprue. This condition is common in travelers to endemic tropical areas and is believed to be caused by an infection. After treatment with folate and antibiotics, patients rapidly improve. Patients are at risk for recurrent disease if they return to the endemic tropical region. Vitamin B12 treatment is also indicated. This patient presents with signs of malabsorption as described by her loss of weight, frequent loose stools, and evidence of multiple vitamin deficiency. She is also hypoalbuminemic and has just returned from a tropical region. There are no risk factors or evidence, by history, for liver disease. The hypoalbuminemia, ecchymosis, and INR are explained by the malabsorptive process instead, which is consistent with a vitamin K deficiency. The findings are consistent with a tropical sprue. These patients often develop a vitamin B12 deficiency, as the terminal ileum is also affected more severely in the proximal small bowel.

A 25-year-old man presents with weight loss, abdominal pain, and bloody diarrhea. Sigmoidoscopy/colonoscopy reveals mucosal erythema and ulceration extending in a continuous fashion proximally from the rectum. Which of the following pathologic findings would also be characteristic of this patient's illness?

You are being tested here on your ability to distinguish between ulcerative colitis (UC) and Crohn's disease. First, you need to figure out which one this patient has. Key clues are the bloody diarrhea (much more common in UC), the rectal involvement, and especially the continuous nature of the mucosal damage. Once you have identified UC, you need to identify the answer choice that is characteristic of UC. The correct answer is pseudopolyps, which are inflammatory polyps found in ulcerative colitis and not Crohn's disease. These post-inflammatory polyps (also known as pseudopolyps) are considered to be high risk for the development of colon cancer, because they are markers of significant inflammatory bowel disease. Features of Crohn's: bowel wall thickening, cobblestoning of mucosa, fistula, transmural lesions

A 77-year-old woman presents to the emergency department with 12 hours of passing bright red blood from her rectum in increasingly large amounts. She becomes dizzy on standing and appears pale. Her blood pressure is 112/60 mm Hg on laying supine and 90/56 mm Hg on sitting upright with legs dangling over the side. Abdominal examination is normal. Which of the following is the most likely diagnosis?

A communication between an arteriole and venule in the cecum is a description of a vascular ectasia, also known as an arteriovenous (AV) malformation. This is a common cause of painless colonic bleeding in the elderly and may present with acute gastrointestinal bleeding (as in this case), chronic gastrointestinal bleeding, or iron-deficiency anemia. These lesions may be difficult to demonstrate, as the bleeding may be intermittent or the colon may be so full of blood that the site of origin is obscured. Techniques used to demonstrate bleeding AV malformations include colonoscopy, intraoperative endoscopy, and visceral angiography. Treatment of these lesions is problematic, because many patients will subsequently develop new or recurrent bleeding vessels. AV malformation or angiodysplasia usually occurs in patients older than age 65 years. It produces intermittent, mild, or severe episodes of painless hematochezia. Significant bleeding may cause orthostatic hypotension, tachycardia, and low blood pressure. Diagnosis may be made via colonoscopy, which identifies these malformations. Treatment during colonoscopy can be with bicap therapy or injection of the vessel with a sclerosing agent.

Four weeks after a camping trip, a 16-year-old boy begins to pass foul-smelling stools. He also develops anorexia and flatulence. None of his friends from the trip are ill. He is generally healthy and takes no medications. His temperature is 37.3ºC (99.1ºF). Physical examination shows diffuse abdominal pain and distention and guaiac-negative stool. The remainder of the examination is unremarkable. A microscopic image of his stool is shown. Which of the following is the most correct statement about his condition?

A 5- to 7-day course of metronidazole (Flagyl) has a cure rate of 80 to 95%. Giardiasis is an enteric infection caused by a flagellated protozoan parasite, Giardia lamblia. It is one of the most commonly identified intestinal parasites in the United States, though it is more common in the developing countries. Campers and those who drink fresh water sources are most likely to acquire this disease. Infection requires the oral ingestion of cysts, with as few as 10 cysts resulting in disease. Incubation is from 3-25 days. After the cysts are ingested, trophozoites are released into the small bowel. These trophozoites are converted to the infectious cysts in the large intestine. First-line therapy is metronidazole (Flagyl) or tinidazole (Tindamax), which has the advantage of a single 2-g curative dose. If treatment fails, a different class of medication should be tried, such as albendazole (Albenza). Parasites are classically cleared from the stool in 3-5 days and symptoms clear in 5-7 days. Health care providers need to stress the necessity of good hand washing after toileting or before preparing food to prevent spread of this disease. Patients need to avoid drinking untreated water, and any raw food that is going to be consumed should be washed thoroughly with uncontaminated water. When traveling to endemic areas, raw food should be avoided and all water boiled for at least 1 minute before consuming.

A 63-year-old man has had mid-abdominal colicky pain with nausea, anorexia, and vomiting for the past 48 hours. The patient has a history of deep venous thrombosis and had a pulmonary embolus in the distant past, but he is not currently on anticoagulants. His temperature is 37ºC (98.6ºF), blood pressure 95/60 mm Hg, and pulse 102/min. On physical examination he has a distended and tympanitic abdomen, but he does not have an "acute abdomen." Abdominal radiographs show multiple distended loops of small bowel and distention of the right colon up to the middle of the transverse colon. Stool is positive for occult blood. Because of his history, a diagnosis of possible mesenteric venous thrombosis is entertained. Which of the following is the most appropriate next diagnostic step?

A CT scan of the abdomen (or MRI scan, which was not offered as a choice) has excellent diagnostic yield when looking for mesenteric venous thrombosis. CT scan with contrast or CT angiogram will show bowel wall thickening, bowel dilation, pneumatosis intestinalis, portal venous gas pattern, and occlusion of the mesenteric vasculature. The CT scan will also provide evidence showing the extent of bowel compromise from ischemia. Mesenteric angiography also remains a viable option in the evaluation of a patient suspected of having mesenteric thrombosis. The clinical picture is indeed very suggestive for that condition.

A 31-year-old man describes a burning sensation in the substernal area after eating chocolate, caffeine, or alcohol. The symptoms are exacerbated at night, and he has woken on several occasions from sleep because of coughing. Over-the-counter antacids and H2 receptor antagonists provide little relief. He often takes antacids before dinner. Which of the following is the most likely underlying cause for his coughing?

Acid-Induced Bronchoconstriction. This patient has the typical symptoms of gastroesophageal reflux disease (GERD) and has now developed nocturnal aspiration of acid. This is one of the extra-esophageal complications of acid reflux. It typically occurs at night, when the patient is supine and has a relatively poor gag reflex because of sleep. The other common extra-esophageal symptoms of GERD are laryngitis and hoarseness, as acid reflux inflames the vocal cords.

After passing his physical examination, a 19-year-old army recruit gives urine and blood samples for further testing. Serum analysis yields elevated ALT, HBsAg, anti-HBc, HBeAg, and bilirubin. All other values are normal. What is the hepatitis B status of this recruit?

Active carrier. The presence of elevated ALT, HBsAg, anti-HBc, HBeAg, and bilirubin all point to active hepatitis B. The HBeAg is known as the envelope, and this test parallels the level of active virus replication and predicts the level of hepatitis B infectivity. Patients who have hepatitis B will have measurable markers in their blood that will determine the status of the patient who has regard to infectivity. Patients who are successfully immunized against hepatitis B will have +HBsAb. Patients who have recent infection will develop HBsAg. If the patient becomes infected with HBV and goes on to clear the infection (spontaneously or with treatment), the patient will undergo a change from +HBsAg to negative status with development of +HBsAb. True infection will cause +HBcAb formation, first with IgM (recent) followed by IgG becoming positive. The IgM core antibody may be the only marker while the patient is recovering from the hepatitis B infection, and this is known as the core window. Vaccination will not cause any reaction to the core. HBeAg is a marker of active viral replication and a marker for how potentially infectious the person is if someone had a blood exposure to that patient. Patients who have active carrier states will have +HBsAg, +HBcAb, and +HBeAg, with this pattern showing active viral replication.

A 23-year-old professional basketball player presents to his health care provider 3 hours before game time complaining of abdominal pain. The symptoms began approximately 8 hours ago in a diffuse fashion. Two hours later, he began feeling nauseated and vomited twice. Over the past 4 hours the abdominal pain has become more severe and well-localized in the right lower quadrant. His examination now reveals well-localized pain in the right lower quadrant inferolateral to the umbilicus. Which of the following is the most likely diagnosis?

Acute appendicitis is one of the most common surgeries performed in the United States and the most common surgical emergency in every decade of life. Although it can occur at any age, many patients are teenagers and young adults. This patient's presentation is typical for acute appendicitis, with initially poorly localized pain followed by nausea and vomiting. In classic appendicitis the pain shifts to the right lower quadrant, where it becomes more localized in the area known as McBurney point. In most patients acute obstruction of the appendiceal orifice by a fecalith initiates the acute appendicitis. Acute appendicitis is caused by obstruction of the appendiceal orifice by a fecalith. It initially presents as acute abdominal pain, which may then become localized to the right lower quadrant (classic appendicitis), accompanied by nausea and vomiting. Appendicitis is usually a clinical diagnosis but may be confirmed with a CT scan if the signs and symptoms are atypical.

A 39-year-old chronic alcoholic comes to the emergency department with complaints of severe epigastric pain, nausea, and vomiting. He admits to actively drinking and reports that the pain is severe and radiates to his back. He reports that he had 3 similar episodes in the past year that required hospitalization and resolved after several days of not eating. His temperature is 38.4ºC (101ºF). Serum amylase and lipase levels are elevated. Which of the following may occur as a complication of this episode?

Adult respiratory distress syndrome due to circulating phospholipase. Acute pancreatitis is suggested by this patient's severe epigastric pain, which radiates to his back and is accompanied by nausea and vomiting. The diagnosis is confirmed by demonstrating serum elevation of the pancreatic enzymes amylase and lipase. More than 80% of the hospital admissions for acute pancreatitis are related to either biliary tract disease (typically with a stone lodging in the duct system below the entry of the pancreatic duct) or alcoholism, and this patient admits to recent active drinking. Acute pancreatitis is a very dangerous disease, not only because it potentially destroys large areas of pancreatic tissue, but also because the destruction of the exocrine gland tissues potentially releases a great many enzymatically or physiologically active substances into the bloodstream. Among the released substances is phospholipase, which circulates through the bloodstream and damages the alveolar capillary membranes in the lungs, predisposing for adult respiratory distress syndrome (ARDS). Acute pancreatitis leads to production of inflammatory mediators that can lead to damage of the alveolocapillary membrane of the lung, which can lead to destruction of the pneumocytes and decrease in the production of surfactant. This leads to increased surface tension in the lung, along with inadequate oxygenation. Bilateral lung infiltrates and potentially ARDS can develop. These patients may require mechanical ventilation if ARDS develops.

A 41-year-old gas station attendant is shot once with a .38-caliber revolver. The entry wound is in the left midclavicular line, 2 inches below the nipple. There is no exit wound. He is hemodynamically stable. Chest radiograph shows a small pneumothorax on the left and demonstrates the bullet to be lodged in the left paraspinal muscles. In addition to the appropriate treatment for the pneumothorax, which of the following will this patient most likely need?

Although this vignette describes a gunshot wound of the chest, we must remember that the chest and the abdomen are not stacked up like pancakes. There is a dome - the diaphragm - that separates them, and thus an area where chest and abdomen overlap. Any gunshot wound below the nipples involves the abdomen, and such is the case here. The management of all gunshot wounds of the abdomen requires exploratory laparotomy. The surgical exploration will identify vascular sites of injury along with any visceral damage, with correction occurring at the time of the exploration.

A patient with a history of hypertension calls his health care provider's office for advice. He has had long-standing heartburn and recently consulted with a gastroenterologist. He underwent an endoscopy and was told that "Barrett mucosa" was found by biopsy. The patient has read in the newspaper that people with this condition will probably develop esophageal cancer. Which of the following is the most appropriate response to this concern?

Barrett's esophagus may occur in a small number of patients who have GERD. This condition is a metaplasia of the normal squamous mucosa of the esophagus to a columnar (glandular) type of epithelium, and is usually seen after repeated acid exposure to the distal esophagus. Tobacco and alcohol use are also thought to contribute to the process but not to the same extent that they contribute to squamous cell esophageal cancer. This condition is most commonly seen in middle-aged Caucasian men who have a long-standing history of reflux. The significance of Barrett's esophagus is that it may lead to the development of low- or high-grade dysplasia, or esophageal adenocarcinoma. This is a very infrequent occurrence when considering the large number of patients who have GERD, however, and even those who have Barrett's esophagus. Barrett's esophagus usually does not resolve with either medical or surgical therapy. Endoscopic surveillance (with multiple small biopsies, because dysplasia cannot be reliably evaluated by endoscopic appearance alone) every 1-2 years has been often recommended, but some studies suggest that it may not be cost effective.

A 28-year-old man with a 15-year history of ulcerative colitis and primary sclerosing cholangitis has recent worsening of his jaundice. His symptoms of ulcerative colitis have been in remission for the past year. He now complains of the onset of steatorrhea approximately 3 months ago and a 12-pound weight loss during that time. Which of the following would most likely account for this patient's recent symptoms?

Bile Salt Deficiency. This patient has a history of ulcerative colitis complicated by primary sclerosing cholangitis. He has developed steatorrhea as the result of poor delivery of bile salts into the small intestine. This occurs because of intrahepatic and extrahepatic duct strictures of the biliary tree, which prevents adequate delivery of bile salts to the small bowel. In the absence of adequate bile salt, maldigestion of fats occurs, because micelle formation does not occur.

A 66-year-old man comes to the urgent care clinic with progressive jaundice, which he first noticed 6 weeks ago. He has never had any chronic medical conditions and takes no medications. He has smoked 1 pack of cigarettes a day for 30 years and drinks a glass of wine each evening. His total bilirubin is 22 mg/dL (normal <1.9 mg/dL), with a direct (conjugated) bilirubin 16 mg/dL (normal <0.3 mg/dL). Transaminases are minimally elevated, whereas his alkaline phosphatase is about 6 times the upper limit of normal. Sonogram shows dilated intrahepatic ducts, dilated extrahepatic ducts, and a very distended, thin-walled gallbladder without stones. Which of the following is the most appropriate next step in diagnosis?

CT scan of upper abdomen. Obstructive jaundice is evident by the high alkaline phosphatase and the dilated biliary ducts. Malignancy is suggested by the dilated, thin-walled gallbladder. If there is a cancer of the head of the pancreas, CT scan has a good chance of showing it in a noninvasive manner. Cancer of the gallbladder or head of the pancreas should be suspected in the setting of obstructive jaundice. Obstructive jaundice is suspected in the setting of elevated direct bilirubin, elevated alkaline phosphatase, and normal transaminase levels.

A 35-year-old woman complains of increasing weakness and shortness of breath. A complete blood count with differential indicates a megaloblastic anemia. Her vitamin B12 level is found to be low. She is also found to have hypothyroidism and diabetes and is given the diagnosis of chronic type A gastritis. Which of the following is associated with this illness?

Chronic type A gastritis is immunologically mediated and is associated with an elevated level of parietal cell antibody. Acid secretion is thus reduced. Patients who have autoimmune disease will often have more than one abnormality associated with these conditions. Vitamin B12 deficiency can be caused by pernicious anemia, which has antibodies to parietal cells located in the stomach. Pernicious anemia results in achlorhydria with decreased stomach acid secretion. Chronic vitamin B12 deficiency caused by pernicious anemia will result in megalolastic anemia. Chronic NSAID use will lead to type B gastritis as well, involving the antrum of the stomach.

A 53-year-old woman comes to the health care provider for an annual examination. She has no complaints. She has hypertension, for which she takes a thiazide diuretic, but no other medical problems. Her past gynecologic history is significant for normal annual Pap tests for many years, her last being 2 months ago. A recent mammogram was negative. Heart, lung, breast, abdomen, and pelvic examinations are unremarkable. Which of the following procedures or tests should most likely be performed on this patient?

Clonoscopy. Cancer screening should be an essential part of an annual examination. Colorectal cancer is a major cause of serious morbidity and mortality for women in the United States: greater than 50,000 new cases are diagnosed each year, and there are more than 25,000 deaths from colorectal cancer. Screening should begin at age 50 in asymptomatic women who have no significant family history. Screening options for colorectal cancer include home fecal occult blood test (FOBT), flexible sigmoidoscopy, the combination of home FOBT and flexible sigmoidoscopy, colonoscopy, and double-contrast barium enema.

A 54-year-old man comes to the health care provider for a periodic health examination. His family history is significant for his mother who died of a cerebrovascular accident at age 72, his father who died of a myocardial infarction at age 68, and his brother who developed sigmoid cancer at age 60. The patient is on no medications except for aspirin, 81 mg daily. Physical examination is unremarkable. The patient asks for a recommendation regarding current cancer screening. Which of the following is the most appropriate screening test for this patient?

Colonoscopy. Any patient who has a first-degree relative who has developed an adenoma or colorectal cancer should undergo colonoscopy for screening at age 50, or 10 years before the relative developed the adenoma or carcinoma, whichever comes first. This patient has a brother who has a colon cancer at age 60; therefore, a full colonoscopy is warranted. Screening recommendations also recommend screening for colon cancer beginning at age 50. Although there are various opinions regarding appropriate screening in the "average risk individual," there is a consensus that full colonoscopy is required in patients who have an increased risk

A 51-year-old man undergoes a barium enema as a colon cancer screening examination. A 3-mm polyp is found in the ascending colon. It is rounded and smooth and meets all the radiographic criteria of a benign sessile polyp. No other lesions are seen within the colon. Which of the following is the most appropriate management?

Colonoscopy. The finding of a sessile mass in the colon, even though it is small and has a benign appearance, mandates a colonoscopy for its removal. This would allow a complete excisional biopsy and histologic review. Although frank carcinoma is unlikely with a mass this small, either a tubular adenoma or a villous adenoma is conceivably present and should be removed both to establish the diagnosis and to prevent the lesion from eventually progressing to cancer. In addition, the structure of the bowel is sufficiently complex that it is not uncommon for other small polyps to be present that were not picked up radiologically. These should also be excised, and their histology reviewed. The need for another diagnostic test to be performed highlights the advantage of performing the colonoscopy as the initial screen rather than barium enema. Colon cancer typically has a precursor lesion that begins as a colonic polyp. The majority of colorectal cancers arise in adenomas. All types of adenomas may be dysplastic and premalignant, with adenomas having a villous component being the most likely to become malignant. When polyps are identified, they should be removed to prevent them from becoming malignant. Colonoscopy is the procedure of choice to remove these polyps.

A patient presents to the emergency department with acute abdominal pain. Which of the following descriptions of the patient's pain would be consistent with a diagnosis of diverticulitis?

Diverticulitis commonly produces a steady, aching pain, localized to the left lower quadrant of the abdomen, with referral to the back in some cases. In addition, a tubular mass (caused by inflammation) may be appreciated on abdominal examination, and the patient may be febrile with an increased white count. Remember, if a patient presents with symptoms and signs similar to appendicitis, but the complaints concern the left rather than the right side of the abdomen, think of diverticulitis as a likely diagnosis. Also, take note that diverticulosis, not diverticulitis, is the most common cause of massive lower gastrointestinal bleeding in adults. Symptomatic diverticulitis presents with fever, leukocytosis, and left lower quadrant pain. It is caused by overactivity of colonic smooth muscle that results in herniation of mucosa and submucosa through the muscle layers of the colon, which form a diverticulum.

A 54-year-old man, who 5 years ago underwent laparotomy for a gunshot wound to the abdomen, is admitted to the hospital because of protracted vomiting and progressive abdominal distention. The symptoms began 5 days ago, and since then he has not had a bowel movement or passed any gas. At the time of hospitalization, he has hyperactive bowel sounds and some abdominal discomfort but he does not have an acute abdomen. Abdominal radiographs show dilated loops of small bowel, multiple air-fluid levels, and no free air under the diaphragms. He is placed on nasogastric suction and intravenous fluids. After 6 hours he develops fever, leukocytosis, and abdominal tenderness. There is severe tenderness when external pressure is applied to his abdomen and then suddenly released. There are no audible bowel sounds. Which of the following is the most appropriate next step in management?

Emergency Exploratory Laparotomy. This patient presents with mechanical intestinal obstruction caused by adhesions which developed as a result of previous exploratory laparotomy, and he now has signs of bowel strangulation. If the strangulated loop is still viable, it has to be freed immediately. If it is necrotic, it has to be resected with equal urgency to prevent continued peritoneal soiling. This patient has now developed an acute abdomen, as manifested by rebound tenderness, fever, and leukocytosis. The clinical picture is typical of bowel obstruction; abdominal distention, vomiting, and hyperactive bowel sounds are common manifestations. The most common cause of bowel obstruction is adhesions from a previous surgery. Regardless of the cause, it is always a surgical emergency.

A 62-year-old social worker with a past history of recurrent GERD, complains of increasing difficulty with swallowing. She has had trouble with solid foods and senses a discomfort in the mid-lower chest after eating meats or dry bread. She has not lost any weight and denies any other medical problems. There is no family history of gastrointestinal malignancy. Physical examination is unremarkable. Which of the following would be the most appropriate next step in the evaluation of this patient's symptoms?

Esophagoscopy. This patient is presenting with symptoms of a peptic stricture after many years of gastroesophageal reflux disease (GERD) symptoms. An endoscopy will allow evaluation of the stricture and biopsy to ensure that the condition is benign. Furthermore, it will allow dilatation of the stricture using an endoscopic-guided balloon to relieve the patient's symptoms.

A 22-year-old man is diagnosed with Crohn's disease limited to the terminal ileum. His symptoms of mild right lower quadrant pain and postprandial diarrhea resolve after the initiation of treatment with mesalamine. Two years later, he develops recurrent episodes of abdominal distention, nausea, and vomiting after large meals. On two occasions, these symptoms are accompanied by inability to pass flatus or bowel movements. Which of the following has this patient most likely developed?

Fibrosis and a stricture in the terminal ileum.This patient who has Crohn's disease has developed symptoms of a small bowel obstruction, a common and important complication of Crohn's disease which occurs as a result of chronic transmural inflammation. It both partially destroys the normal bowel wall and constricts it with thick bands of fibrosis. Other important intestinal complications can include fistula formation and chronic abscesses. In addition, a wide variety of extraintestinal complications can include autoimmune diseases (arthritis, aphthous ulcers, erythema nodosum, pyoderma gangrenosum, eye involvement, ankylosing spondylitis, primary sclerosing cholangitis) and complications related to disrupted bowel physiology (renal complications, including kidney stones and urinary tract obstruction, malabsorption, and amyloidosis secondary to long-standing inflammation). A fistula from the ileum to the sigmoid can develop in patients who have Crohn's disease, but it will present with symptoms of diarrhea (because of the bypass of a large portion of the colon) and not obstruction.

A 32-year-old woman has had a 15-year history of heartburn. Over the past 4 months, she has had difficulty swallowing large bites of solid food. She has no difficulty with soft foods or liquids, and she has not lost weight. Which of the following is the most likely explanation for her symptoms?

Fibrosis and narrowing at the distal esophagus. This patient has classic symptoms of mechanical dysphagia, as she has difficulty with large solid food but not softer foods or liquids. Mechanical dysphagia frequently follows many years of heartburn and is often indicative of a peptic stricture that has developed as a result of fibrosis after a long period of chronic inflammation caused by gastroesophageal reflux disease (GERD). These benign strictures can usually be dilated endoscopically. An intensive regimen of proton pump inhibitors should then be instituted to reduce the frequency of recurrence. This patient should be evaluated endoscopically to rule out Barrett's esophagus, which may occur with long-term untreated GERD.

A previously healthy 7-year-old girl comes to the office with complaints of episodic abdominal pain over the past several months. The pain is periumbilical and sharp but does not wake her from sleep and rarely interferes with play. She has no fever, joint complaints, or constipation or diarrhea. Growth and development have been normal. Physical examination is within normal limits. Which of the following is the most likely diagnosis?

Functional abdominal pain is pain that lasts for more than 3 months and rarely interferes with normal activity. The pain is periumbilical and often hard to describe. The pain typically does not awaken patients from sleep or interfere with pleasant activities. The pain is real and is the result of the regulation of gastrointestinal motility in response to either psychologic or physical stress. Functional abdominal pain is seen most frequently in girls 8-12 years of age, and patients who have this disorder typically complain of vague, persistent, central abdominal pain that can be associated with nausea and/or vomiting. Examination will reveal periumbilical tenderness with a soft abdomen and no rebound or guarding. The remainder of the physical examination is normal. This is a diagnosis of exclusion and no red flag symptoms are present (weight loss, failure to thrive, nocturnal diarrhea, signs of bleeding).

Six days after an uncomplicated left nephrectomy, a 42-year-old man complains of moderate abdominal pain and diarrhea. He is stooling approximately 6 times a day with profuse watery diarrhea. The patient also experiences painful cramps during these episodes. In other medical history, the patient has a history of hypertension and elevated cholesterol. His medications include metoprolol (Lopressor) and simvastatin (Zocor). He denies any recent cardiopulmonary history and any history of chronic gastrointestinal disorders. His chart indicates that he received a 2-day regimen of cefazolin (Ancef) for perioperative skin prophylaxis. His temperature is 38.3ºC (101.0ºF). Stool is positive for fecal leukocytes and occult blood. The diagnosis is established with additional studies and he is given treatment. Unfortunately he fails to respond to therapy and requires surgery. An image of a segment of his colon is shown. Which of the following is the most likely pathogen?

This patient is experiencing C. difficile colitis, also known as pseudomembranous colitis. This type of diarrhea is common in hospitalized patients; at least 30% of hospitalized patients are colonized with the organism. Certain antibiotics have a predilection for causing C. difficile colitis: clindamycin, ampicillin, amoxicillin, and cephalosporins have a high incidence of causing the diarrhea. These antibiotics suppress normal gut flora, allowing Clostridium to proliferate. Two endotoxins (A and B) cause the symptoms and the colonic pathology noted in the vignette. Diagnosis is made by ELISA test for endotoxin A and B; sigmoidoscopy also provides additional diagnostic information, particularly the presence of plaques (also called pseudomembranes). Treatment is oral metronidazole or vancomycin. Surgery is sometimes indicated for recurrences that are unresponsive to therapy or when complications such as toxic megacolon occur. The 2 main risk factors for Clostridium difficile colitis are recent antibiotic use and recent hospitalization. Patients who have this pathogen typically present with diarrhea, abdominal pain, and leukocytosis. There may also be associated fever, abdominal tenderness, and abdominal distention. Treatment involves discontinuing the antibiotic and administering oral metronidazole (Flagyl) or vancomycin.

A 28-year-old man with end-stage renal disease (ESRD) on continuous ambulatory peritoneal dialysis (CAPD) for 2 months is brought to the emergency department with fever, abdominal pain, and cloudy dialysis fluid. The pain has been present for 12 hours. There is no diarrhea or vomiting. The patient has ESRD secondary to chronic glomerulonephritis; there is no history of diabetes, urinary infections, or antibiotic use. Examination reveals a temperature of 38.9ºC (102ºF) and blood pressure 110/70 mm Hg. The throat is clear, as are the lungs. Cardiac examination reveals a grade 2/6 systolic murmur. Abdominal examination reveals decreased bowel sounds with diffuse tenderness. There is mild rebound. There is no edema or skin rash. A complete blood count shows a leukocyte count of 14,200/mm3 (normal 5,000-10,000/mm3) and hemoglobin 12.5 g/dL (normal 13.8-17.0 g/dL). Peritoneal fluid is cloudy with 1,000 white blood cells, 85% of which are polymorphonuclear leukocytes. Gram stain of the peritoneal fluid is negative. Cultures of blood and peritoneal dialysis fluid are taken. Which of the following is the most appropriate initial step in management?

Gram-negative organisms classically cause peritonitis in a patient on CAPD, but there is a recent tendency for this condition to be caused by gram-positive pathogens such as Staphylococcus aureus or Epidermidis. It is usually characterized by abdominal pain and over 100 white blood cells (typically polymorphonuclear leukocytes) in a sample of peritoneal dialysis fluid. Intravenous ceftriaxone and vancomycin would be a reasonable treatment to cover both gram-negative and gram-positive pathogens.

A previously healthy, intoxicated 19-year-old man is driving a car without using a seat belt. He crashes the car into the back of a parked truck. In the process he slams his abdomen into the steering wheel and ruptures his spleen. Which of the following is the most important problem associated with this type of injury?

Internal Blood Loss. The spleen is a highly vascularized organ and is vulnerable to traumatic rupture. This can occur "spontaneously" (i.e., with minimal trauma, such as falling against a table or even overly vigorous palpation during a physical examination) in patients who have an enlarged spleen caused by disease (e.g., leukemia, mononucleosis, autoimmune diseases with red blood cell sequestration in the spleen, or as a complication of portal hypertension). Alternatively, splenic rupture can occur in previously normal individuals who have severe trauma to the abdomen. In either case, the heavily vascularized spleen is usually unable to stop (often massively) bleeding internally. Emergency splenectomy is indicated to control the bleeding.

A 57-year-old woman with a 30-year history of alcoholism and liver disease visits her health care provider complaining of abdominal swelling and shortness of breath. The health care provider determines that she has severe ascites. Which of the following factors contributes to the accumulation of fluid in the abdominal cavity?

Increased hydrostatic pressure in splanchnic capillary beds. Ascites often occurs in patients who have cirrhosis and other forms of severe liver disease and is usually noticed by the patient because of abdominal swelling. Shortness of breath may occur because the diaphragm is elevated when the accumulation of fluid becomes more pronounced. Portal hypertension plays an important role in the production of ascites by raising capillary hydrostatic pressure within the splanchnic bed. Elevated serum levels of epinephrine and norepinephrine occur as a result of increased central sympathetic outflow, in patients who have cirrhosis and ascites. The increased sympathetic output leads to decreased natriuresis by activation of the renin-angiotensin system and diminished sensitivity to atrial natriuretic peptide. Interstitial fluid often weeps freely from the surface of the cirrhotic liver because of distortion and obstruction of hepatic lymphatics with resultant decreased hepatic flow. This interstitial fluid has a high protein concentration because the endothelial lining of the hepatic sinusoids is discontinuous. The entry of protein-rich interstitial fluid into the peritoneal cavity may account for the high protein concentration present in the ascitic fluid of some patients. The high protein content of the interstitial fluid results in fluid retention and an increase in plasma volume. Ascites is the accumulation of fluid in the peritoneal cavity; the causes are usually related to cirrhosis. With cirrhosis the liver undergoes loss of functional cells, causing increased portal pressures and increased pressures in the gastric and splenic venous systems. Other causes of ascites include hypoalbuminemia (generalized swelling caused by poor oncotic pressure in the cell with resultant cellular leakage) and neoplasms such as ovarian cancer with malignant accumulation of fluid in the peritoneal cavity.

A male patient has a sudden onset of fretfulness and pain. He curls up with his legs located in a fetal position. Over the next few hours he alternates between episodes of pain/crying with tears and acting normally. The patient's mother fears something is terribly wrong and brings him to the emergency department. His past medical history is unremarkable. The previous week he had had a cold with a runny nose. Stools had been normal that day but at the hospital he has a semi-soft stool with some blood and mucus. On examination the patient is quiet. The abdomen is surprisingly soft and normal. This classic presentation of intussusception is most likely to occur in which of the following age groups?

Intussusception usually occurs between ages 3 and 12 months, with the peak incidence at 6 months of age. It characteristically presents with episodes of distress and crying interspersed with quiet periods of normal behavior and playing. More than 50% of children will pass stool mixed with mucus and blood, classically known as the "currant jelly" stool. Usually the abdomen is soft and nontender, but with advanced intussusception there may be signs of peritonitis. Sometimes a sausage-like mass may be palpable in the upper abdomen. A barium enema is performed for the diagnosis and treatment of the intussusception.

A 69-year-old woman comes to her health care provider's office complaining of 1 week of crampy lower abdominal pain and bloody diarrhea. She had previously been followed for symptoms of stable exertional angina and hypertension. She had an uncomplicated myocardial infarction 3 years earlier. Her symptoms began 1 week ago with mild postprandial abdominal cramping followed by diarrhea, which became bloody after 2 days. She has not traveled recently and is a retired librarian. Her temperature is 38.6ºC (101.4ºF), blood pressure 120/84 mm Hg, and pulse 96/min. She has moderate tenderness to palpation of the left lower quadrant. Rectal examination reveals bloody stool and no masses. Which of the following is the most likely diagnosis?

Ischemic Colitis. This elderly woman who has a history of atherosclerotic vascular disease as demonstrated by a history of a myocardial infarction and exertional angina has developed colitis symptoms, as demonstrated by the left lower quadrant pain and bloody diarrhea. This is typically caused by ischemia of small branches of the inferior mesenteric artery. The bleeding is caused by poor blood flow to the bowel, which leads to mucosal sloughing that can cause blood loss into the bowel lumen. The diagnosis is suspected clinically and generally confirmed with a flexible sigmoidoscopy, because many cases involve the rectosigmoid region.

A 17-year-old boy is brought to the health care provider by his mother because "he has been sick for some time, and the health care providers do not seem to know what is wrong with him." The precipitating event for their visit today was the fact that the boy's sclera turned yellow a few days ago. On further questioning a history of protracted diarrhea is elicited. For about 3 years the patient has had intermittent episodes of crampy abdominal pain and diarrhea. On one occasion he almost had an appendectomy because of right lower quadrant abdominal pain; surgery was canceled when he improved with observation and a sonogram had been read as negative for acute appendicitis. At this time he has a bilirubin of 3.6 mg/dL (normal <1.9 mg/dL) with 2.9 mg/dL conjugated (normal <0.3 mg/dL), alkaline phosphatase 625 U/L (normal 44-147 u/L), and near-normal transaminases (AST, ALT levels). Sonogram of the right upper quadrant shows normal caliber extrahepatic ducts, no gallstones, and no evidence of liver abscess. Stool cultures are negative. He has never been out of the country. Which of the following is the most likely diagnosis? acute hepatitis, ameobic abscess of liver, cancer of head of pancreas, ischemic colitis, sclerosing cholangitis associated with IBD

Jaundice with very high alkaline phosphatase is consistent with cholangitis. Cholangitis is typically from obstruction or cholestasis, but sonogram does not show dilated extrahepatic ducts. Normal caliber extrahepatic ducts without gallstones or gallbladder sludge findings are most consistent with sclerosing cholangitis. Furthermore, the current liver findings must be in some way related to his intermittent diarrhea, and sclerosing cholangitis is seen in conjunction with inflammatory bowel disease, which in turn is a perfect bet for intermittent, crampy diarrhea in a 17-year-old. The major risk factors for sclerosing cholangitis are male sex plus history of inflammatory bowel disease. There is also an increased risk for first-degree relatives of patients who have primary sclerosing cholangitis. Serum alkaline phosphatase levels can be elevated up to 3 times normal. The transaminase levels show only mild to moderate elevation in patients who have sclerosing cholangitis. Conjugated bilirubin levels will also be elevated, which pushes the total bilirubin levels to also be elevated. MRCP or ERCP may show intrahepatic and/or extrahepatic strictures and dilation with or without a prominent biliary stricture.

A 23-year-old woman seeks help for exquisite pain with defecation and blood streaks on the outside of her stools, which she has been having for several weeks. Because of the pain, she has avoided having bowel movements, and when she finally did, the stools were hard and even more painful. When seen, she has no fever or leukocytosis. Physical examination has to be done under spinal anesthesia, because the patient was so afraid of the pain that she initially refused even inspection of the area. The examination confirms the suspected diagnosis, and she is placed on stool softeners and appropriate topical agents, but without success. She is now willing to undergo more aggressive treatment. Which of the following is the most appropriate next step?

Lateral internal sphincterotomy. The clinical picture is classic for anal fissure, which is a split in the skin of the anatl canal. Patients with this condition present with pain with defecation along with rectal bleeding from the area of the split. It commonly occurs in young to middle age persons. Pain is described as severe and this can adversely affect quality of life. If spontaneous healing does not occur, surgical intervention with lateral sphincterotomy can be performed. Anal fissure patients will complain of severe pain with defecation and following defecation with the pain lasting up to several hours following the bowel movement. On physical examination of the anus, there may be marked spasm of the sphincter muscles seen or a split in the skin of the distal anal canal. Conservative therapy is with stool softeners, sitz baths, high fiber diet, adequate fluid intake, and topical anesthetics.

A patient with acute hepatitis B comes to the office complaining of severe fatigue, low-grade fevers, and weight loss. He was diagnosed with hepatitis B two weeks earlier, when hepatitis B surface antigen was positive. Physical examination reveals jaundice with diffuse skin excoriations. Liver and spleen are both markedly enlarged and tender. Peripheral edema is present. Which of the following would be the worst prognostic sign?

Many liver function abnormalities are seen in patients with acute hepatitis B, but the signs of hepatic failure (coagulopathy or encephalopathy) are the worst prognostic signs. Patients may require emergent liver transplantation. Patients with acute hepatitis B will have marked abnormalities in their liver function tests. Because of the amount of inflammation that occurs with the acute hepatitis B infection, one would expect patients to have increased transaminase levels and bilirubin levels. High levels are an expected phenomenon and are related to the acute infection. As the liver fails, liver function is affected. The final sign that the liver is failing is in the area of making clotting factors. When the liver fails to make clotting factors, there is clear evidence that it has reached its end-stage of failure. Prolongation of the prothrombin time therefore gives evidence that the patient is no longer making clotting factors, which signifies end-stage disease and the worst prognosis for patients who have acute hepatitis.

A 56-year-old alcoholic comes to the health care provider complaining of 6 months of worsening midepigastric pain radiating to his back. The pain is exacerbated by eating and continued alcohol use. One month ago he was told that he had developed diabetes. He has also noted that, over the past 2 months, he has had greasy, foul-smelling, large-volume stools. Which of the following is the most likely cause of this man's steatorrhea?

Midepigastric pain radiating to the back and exacerbated by eating and continued alcohol use should suggest the diagnosis of pancreatitis. Significant risk factors for pancreatitis include alcoholism (as this patient has) and biliary tract disease (for which there is no evidence in this patient). It is a little unclear from the history whether the patient is experiencing recurrent episodes of acute pancreatitis or is developing the onset of chronic pancreatitis. Whichever is the case, both processes are capable of destroying enough exocrine pancreatic tissue to predispose for exocrine pancreatic insufficiency with resultant insufficient production of pancreatic enzymes, such as lipase, which normally facilitates fat usage, to allow for normal digestion. Steatorrhea, or excess fat in stool, is seen clinically as greasy, foul-smelling (because of bacterial action on the fat), large-volume (in part because of gas production by bacteria and in part because of failure to digest most of the food) stools.

A 24-year-old graduate student is found to have acute myelogenous leukemia (AML). Before undergoing a planned bone marrow transplant, he begins aggressive multi-drug chemotherapy. His tolerance of this regimen is severely limited, however, by intractable vomiting with each chemotherapy cycle. Which of the following will most likely reduce vomiting in this patient?

Ondansetron (Zofran), a 5-hydroxytryptamine3 antagonist, is the most potent antiemetic available for chemotherapy-induced vomiting. It is safe and is available as an oral (including dissolving tablet) or IV medication. Ondansetron has greatly improved the symptoms of many patients who were previously unable to tolerate full-dose chemotherapy because of severe vomiting. It has side effects only infrequently, the most common being constipation. 5-HT3 receptor antagonists are extremely effective anti-emetic agents used for prevention and treatment of chemotherapy-induced emesis. Their mechanism of action is that they selectively antagonize serotonin 5-HT3 receptors. Examples include dolasetron (Anzemet), granisetron (Kytril), and ondansetron (Zofran).

A 4-month-old boy has gained only 10 ounces since birth. He has failed to gain weight with multiple different formula preparations. His stools have been loose and fatty. An older sister had similar symptoms and has been repeatedly hospitalized for failure to thrive and recurrent pulmonary infections. Which of the following is the most likely cause of this patient's gastrointestinal symptoms?

Pancreatic exocrine insufficiency. The patient described here has cystic fibrosis. This inherited disease primarily affects the gastrointestinal and respiratory systems. Cystic fibrosis can present in early infancy with meconium ileus, in the first year of life with steatorrhea and failure to thrive, or in older childhood with recurrent pulmonary infections. The pancreatic secretions are characteristically very thick and tend to occlude the pancreatic duct system. Steatorrhea has developed in this infant because of pancreatic duct inspissation of secretions and the resultant decrease in delivery of pancreatic enzymes to the small bowel. Cystic fibrosis is a genetically inherited condition that affects multiple organs, especially the GI and respiratory tracts. There is a defect in the salt and water transport across the epithelial surfaces that results in pancreatic dysfunction resulting in calorie malabsorption. There is also lung disease from mucus retention, infection, and inflammation. Pancreatic enzymes are given to support the patient's growth and nutrition.

A 24-year-old man comes to the student health clinic with complaints of chronic epigastric pain 1 week before taking final exams. The patient has been awakened from sleep by epigastric pain at approximately 3 AM on several occasions. Ranitidine (Zantac) has improved his symptoms transiently. Eating generally improves his symptoms for about 30 minutes, but it is then followed by increased pain. Which of the following is the most likely cause of his symptoms?

Peptic ulcer disease is strongly suggested by this patient's chronic epigastric pain that is severe enough to awaken him at night and is temporarily relieved with ranitidine and eating. In the absence of nonsteroidal anti-inflammatory drug (NSAID) use, the most likely etiology is antral colonization with Helicobacter pylori. This gram-negative organism lives in the mucous layer that lines the stomach and apparently uses the mucus as its food source. The result is focal disruption of the mucosal barrier, predisposing for both chronic gastritis and gastric peptic ulcer formation. In the past, medical personnel tended to blame ulcer formation on "stress," with the implication that correcting the problem involved teaching the patient to be more calm. Now that we more fully understand peptic ulcer disease, we treat the bacterial colonization as part of our primary therapy.

A 40-year-old woman presents with intense pruritus and fatigue. Her medical history is significant for hypothyroidism. Physical examination reveals a slightly enlarged liver and xanthomas. Laboratory studies show a cholesterol level of 538 mg/dL and alkaline phosphatase 571 IU/L. Which of the following serum values would most likely be elevated?

Primary biliary cirrhosis is an autoimmune disease associated with elevated antimitochondrial antibodies. Hashimoto thyroiditis, an autoimmune disease that is the most common cause of hypothyroidism, is also associated with increased levels of this antibody. Primary biliary cirrhosis is a condition seen primarily in women in their fifties. Presentation is consistent with cholestatic liver disease, with pruritis and fatigue being common presenting symptoms. Disease progression is controlled with the use of corticosteroids and ursodiol. Liver transplantation is an effective therapy for end-stage liver disease. This is an autoimmune disease of the small intrahepatic bile ducts that results in portal tract inflammation. Alkaline phosphatase and bilirubin, markers of cholestasis, are elevated. Ninety percent of affected patients will have a positive antimitochondrial antibody, which is considered the serum marker for this condition.

A 45-year-old man with alcoholic cirrhosis is bleeding from a duodenal ulcer. He has required 6 units of blood over the past 8 hours, and all conservative measures to stop the bleeding, including irrigation with cold saline, intravenous vasopressin, and endoscopic use of the laser have failed. He is being considered for surgical intervention. Laboratory studies done at the time of admission, when he had received only 1 unit of blood, showed a bilirubin 4.5 mg/dL (normal <1.9 mg/dL), prothrombin time 22 seconds (normal 11-15 seconds), and serum albumin 1.8 g/dL (normal 3.5-5.5 g/dL). He was mentally clear when he came in but has since developed encephalopathy and is comatose. Which of the following best describes his operative risk?

Prohibitive regardless of attempts to improve his condition. The studies show extremely marginal liver function, which would be tipped into overt liver failure by an anesthetic and an operation. He is not a surgical candidate. The patient has signs of end-stage liver disease with hepatic encephalopathy along with poor manufacturing of clotting factors. Because the patient would need to have anesthetic agents metabolized through the liver, he is incapable of surviving the surgery at this point in time.

A 1-month-old boy is brought to the emergency department by his mother, who states that he has been having forceful vomiting for the past several days. She states that he vomits every time she feeds him, and the situation seems to be getting worse, though he seems in no pain. She describes the vomitus as non-bilious and his stools as normal with no blood. On examination the infant appears to be mildly dehydrated, with soft abdomen and palpable 2-cm, firm, mobile mass in the right upper quadrant. Which of the following is the most likely diagnosis?

Pyloric stenosis typically develops between 3 and 6 weeks of life. It is caused by hypertrophy of the pyloric muscle, which obstructs gastric outflow. The incidence is higher in males and first-born infants. The symptoms include progressively worsening vomiting, which becomes projectile and is non-bilious. On examination peristaltic waves may be seen, and an olive-sized mass is usually palpated in the right upper quadrant. The vomiting continues despite multiple formula changes. Ultrasound is typically used as a diagnostic evaluation tool. Treatment is with IV fluid and electrolyte replacement, followed by pyloromyotomy done via an open or laparoscopic approach.

A 7-year-old boy passes a large, bloody bowel movement. He is hemodynamically stable, with hemoglobin 14 g/dL (normal 13.8-17.0 g/dL). Nasogastric aspiration yields clear, greenish fluid. Physical examination, including anoscopy, is unremarkable. Which of the following is the most appropriate next diagnostic test?

Radioactively labeled technetium scan. In this age group, with no obvious anal pathology and negative gastric aspirate, the leading cause of gastrointestinal bleeding is Meckel diverticulum. The specific source is ulceration of the normal ileal mucosa by acid produced by gastric mucosa in the diverticulum. The technetium scan identifies that ectopic gastric mucosa. Meckel diverticulum is a common cause of lower GI bleeding in the pediatric population. Patients will present with either painless melena or bright red blood per rectum. When Meckel diverticular bleeding occurs, a radionuclide imaging study will show an uptake caused by the technetium binding to ectopic gastric parietal cells. Another test to consider with suspected Meckel diverticulum is mesenteric angiography, which will show extravasation of the dye at the bleeding site in the terminal ileum.

The health care provider becomes concerned when after 30 hours, a newborn infant has not passed meconium. He was full term with a birth weight of 3,856 g (8 lb 8 oz). The pregnancy was uncomplicated. The infant appears well, with no respiratory distress. Slight abdominal distention is noted. Rectal examination reveals a slightly tight rectum and results in a greenish gush of stool. Which of the following tests will probably confirm the likely diagnosis?

Rectal Biopsy. Hirschsprung's disease or congenital aganglionic megacolon is caused by a congenital absence of the ganglion cells of both the Meissner and Auerbach plexuses. It is the most common cause of lower intestinal obstruction in the neonatal period. In early childhood it may present as chronic constipation with intermittent fecal soiling. It occurs predominantly in males and there is an increased family incidence. Surgical treatment is indicated, but the diagnosis is confirmed by a suction biopsy that can be easily performed without general anesthesia. The biopsy specimen would reveal an absence of ganglion cells in the submucosal and myenteric plexuses.

A 5-month-old boy is brought to the office because of frequent watery stools and vomiting present for the past 2 days. The child has been in daycare since age 3 months and has had several episodes of fever and runny nose in the meantime, but this is the first time he has had diarrhea. It started approximately 48 hours earlier with the passage of six to seven loose, watery stools with a foul smell. He had also vomited a couple of times and has had fever since the beginning of the symptoms. The parents had called the daycare to see if other children were affected, but learned that no one else had similar symptoms. They had not traveled anywhere since the child was born. On physical examination, the child is in moderate distress, with a temperature of 38.1ºC (100.6ºF), a pulse of 120/min, and respirations of 26/min. He has dry mucosa and his anterior fontanelle is sunken. Skin turgor is also diminished. Palpation of the abdomen does not reveal tenderness. On auscultation, increased peristalsis can be heard. The stool is guaiac-positive for occult blood but shows no visible gross blood on examination. He is given intravenous rehydration in the office and sent home on supportive treatment with lots of fluids and antipyretics as needed. Two days later, the mother calls the office to say that all the symptoms have completely resolved and the child is feeling well and behaving normally. Which of the following was the most likely cause of this patient's illness?

Rotavirus. Causes of acute and chronic diarrhea are age-dependent. Acute diarrhea is almost always infectious, with gastroenteritis the most common cause in any age group. Food poisoning, systemic infections, parasitic infections, and antibiotics are other causes. Viral agents are the most common cause of acute diarrhea in children, and rotavirus is the most common viral cause of diarrhea in the winter. Enteric adenovirus and Norwalk virus are also common causes of diarrhea. Bacterial causes include E. coli, Salmonella, Shigella, Campylobacter, Yersinia, and Clostridium. Parasitic causes include Entamoebahistolytica, Giardia, and Cryptosporidium. Clinically, rotavirus infection presents with watery diarrhea that can last up to 7 to 10 days. It may be accompanied by 3 to 4 days of vomiting. Fever may be present. The stool may be positive for occult blood. There is no abdominal tenderness associated with rotavirus infection. Viruses account for about 70 to 80% of episodes of acute gastroenteritis in children, with rotavirus being the most common identifiable cause of gastroenteritis in children. Rotavirus is commonly spread in the daycare setting. Rotavirus is ubiquitous and almost all children are affected by 3 years of age. Spread is via fecal-oral route, but recent studies have suggested that rotavirus may be spread by respiratory droplets. Treatment is conservative, with maintenance of hydration being the most important element of care. Supporting the patient's hydration status until the virus is spontaneously cleared is the management of choice. Rotavirus infection can cause simultaneous nausea, vomiting, and diarrhea along with a low grade fever that may be caused by the dehydration that rapidly occurs. Strict hand washing techniques may help to limit the spread of this virulent pathogen. Diarrhea is defined as an increased stool output, with excess loss of fluid and electrolytes. It can be classified as acute or chronic. Several mechanisms exist that cause diarrhea, of which more than one may be present: 1. secretory: decreased absorption, increased secretion. 2. osmotic: maldigestion, transport defects, ingestion of unabsorbable solute. 3. increased motility: decreased transit time. 4. decreased surface area: inflammation, decreased colonic absorption, increased motility.

A 55-year-old HIV-positive man has a fungating mass growing out of the anus. He can feel it when he wipes himself after having a bowel movement, though it is not painful. For the past 6 months he has noticed blood on the toilet paper and from time to time there has also been blood coating the outside of the stools. He has lost weight, and he looks emaciated and ill. On physical examination the mass is easily visible. It measures 3.5 cm in diameter, is fixed to surrounding tissues, and appears to grow out of the anal canal. He also has rock-hard, enlarged lymph nodes on both groins, some of them as large as 2 cm in diameter. Which of the following is the most likely diagnosis?

SCC of anus. The entire description is classic for anal cancer, but the best evidence is the presence of metastasis in the inguinal nodes. The mass that is visible is consistent with an anal mass.

A 54-year-old woman complains of severe lower abdominal pain and distention. The symptoms began approximately 24 hours ago, when her abdomen became visibly swollen and she developed nausea and vomiting. She has not moved her bowels over the past 24 hours. Over the past 4 months, she has lost 14 pounds and has noted progressive symptoms of constipation. On several occasions she has noted blood mixed in with her bowel movements, which have become thinner in caliber. She denies any recent travel, use of antibiotics, or fevers. On physical examination she appears acutely uncomfortable and has a temperature of 38.3ºC (100.9ºF). The abdomen is diffusely distended and tender to palpation in the left lower quadrant. There are hyperactive rushing bowel sounds. On rectal examination the stool is brown and guaiac positive. An obstructive series reveals multiple small bowel air fluid levels and a dilated colon proximal to the sigmoid colon. Which of the following is the most likely diagnosis?

Sigmoid Carcinoma. This patient has symptoms of a chronic gastrointestinal process as demonstrated by her weight loss, change in bowel habits, and thinner caliber stools with bleeding. The thinner caliber stools specifically suggest that a mass lesion or luminal narrowing is present. Of the lesions listed, only colonic cancer would be likely to produce this pattern. In other settings, Crohn's disease and tuberculosis of the colon could produce similar clinical patterns. This patient's change in symptoms over the past 24 hours suggests that she may have developed an acute large bowel obstruction.

A 9-month-old girl is brought to the clinic by her parents because she has an abdominal mass. Physical examination shows an umbilical defect about 1 cm in diameter, with a small bulge when the girl cries. The hernial contents can be easily reduced. The hernia is not painful and the girl is otherwise asymptomatic. She is eating well and reaching all developmental milestones. Which of the following is the most appropriate next step in management?

Small umbilical hernias can close spontaneously up to the age of 2. If they are asymptomatic and pose no immediate risk for strangulation, they should be left alone. Umbilical hernias occur as a result of the umbilical ring failing to close. This defect allows protrusion of the peritoneal sac, which could contain intra-abdominal contents such as the omentum or bowel. Umbilical hernias are the only type of hernia by which spontaneous healing occurs. Most umbilical hernias will close on their own by age 4-5, often correcting themselves as infants develop abdominal musculature by crawling and walking. If the umbilical hernia fails to resolve, elective surgical correction may be considered after age 5 (due to higher risk for incarceration if the hernia persists later in life).

A 44-year-old woman with a history of recurrent biliary colic presents with 18 hours of very severe right upper quadrant pain, fever, and jaundice. An abdominal ultrasound reveals a markedly dilated common bile duct. Multiple gallstones are seen in the gallbladder. Which of the following would best determine whether there is cystic duct obstruction?

This patient has evidence of a prolonged episode of biliary colic, which may progress to acute cholecystitis. An abdominal ultrasound has revealed a markedly dilated common bile duct, as well as gallstones within the gallbladder. Acute cholecystitis is the result of cystic duct obstruction, and this would be best demonstrated by a HIDA scan. The HIDA scan or cholescintigraphy scan will directly show cystic duct obstruction by showing a failure of gallbladder filling with normal hepatic uptake of iminodiacetic acid. For a patient who has no blockage of the cystic duct, the test is normal. This test will also show the function of the gallbladder; some patients do not have cystic duct blockage but rather a failure of the gallbladder to contract. This would make the gallbladder dysfunctional, indicating a need for cholecystectomy.

A 44-year-old school bus driver comes to the emergency department complaining of severe abdominal pain. She reports that the pain began approximately 8 hours ago, after eating lunch at a fast-food restaurant. The pain has become increasingly severe and radiates to her back. She recalls a similar episode, lasting 3 hours, 2 months earlier and another episode, lasting 12 hours, during her last pregnancy. She is afebrile, and without jaundice, and has right upper quadrant tenderness with deep palpation of this area. A rectal examination reveals brown stool that is negative for occult blood. Her leukocyte count is 12,900/mm3, (normal 5,000-10,000) and her hematocrit is 39% (normal 41-50%). Her total bilirubin is 2.1 mg/dL (normal <1.9). An ultrasound is performed at the bedside revealing a positive ultrasound Murphy sign but there were no stones or sludge noted in the gallbladder and the cystic duct is not dilated. Which of the following is the most appropriate next step in her diagnostic evaluation? CT of Abdomen and Pelvis, Endoscopic Retrograde Cholangiopancreatography, Hepatobiliary Iminiodiacetic Acid Scan, Percutaneous Transhepatic Cholangiogram, Upper GI Barium Study.

This patient has the classic presentation of acute cholecystitis. The episodes she had several months ago and during pregnancy suggest a prior history of biliary colic. A HIDA scan is a noninvasive nuclear medicine test that will reveal obstruction of the cystic duct, which is caused by an impacted gallstone and is the cause of acute cholecystitis. The patient's elevated total bilirubin provides evidence that the bile duct is blocked. The HIDA scan can also be used to identify gallbladder dysfunction for patients whose gallbladders do not function but who may not have stones. ERCP is useful for evaluation of the common bile duct but is of a less value in evaluation of the cystic duct and, furthermore, is a far more invasive test than a HIDA scan. Acute cholecystitis is typically caused by an impacted stone in the gallbladder neck or cystic duct. This impacted stone leads to inflammation within the gallbladder wall. Ultrasound is the initial test of choice for evaluation of suspected cholecystitis, with HIDA scan or MRI scan being done when the diagnosis is in question or when gallbladder function is being assessed.

A 44-year-old woman delivers a 3,120 g (6 lb 14 oz) newborn boy. Her pregnancy was normal except that she noted decreased fetal movement compared to her previous pregnancies. She declined an amniocentesis offered by her obstetrician. Physical examination of the newborn reveals an infant who has facial features suggestive of Down syndrome. The infant then has bilious vomiting. A radiograph showing the kidneys, ureters, and bladder (KUB) is performed, which shows a distended and gas-filled stomach and proximal duodenum and the absence of gas in the distal bowel. Which of the following is the most likely cause of the abdominal signs and symptoms?

The "double bubble" sign is pathognomonic for duodenal atresia, which is a congenital anomaly associated with Down syndrome. Two large gas collections, one in the stomach and one in the proximal duodenum are the only radiographic lucencies visible in the GI tract. Duodenal atresia is associated with bilious vomiting. Children who have Down syndrome can also have esophageal atresia, imperforate anus, endocardial cushion defects, and hypotonia. Duodenal atresia may be caused by congenital GI disorders. Patients who have Down syndrome may have duodenal or anal atresia or duodenal or anal stenosis. Patients who have duodenal atresia may have abdominal distention and bilious vomiting. Plain radiographs will show evidence of intestinal atresia with a double collection of air in both the stomach and proximal small bowel.

A 4-year-old girl was diagnosed with left-sided otitis media 10 days ago and prescribed oral amoxicillin 40 mg/kg/day for 7 days. She has since developed watery, bloody, diarrhea with mucus, crampy abdominal pain, and fever. On physical examination her temperature is 39.4ºC (102.9ºF), pulse 88/min, and respirations 16/min. She has normal bowel sounds and is diffusely tender to palpation. Which of the following is the most appropriate initial step in diagnosis?

The child has pseudomembranous colitis, caused by the toxins produced by Clostridium difficile. The colon is colonized by C. difficile after eradication of the normal microflora by a course of antibiotics. Virtually all antibiotics have been implicated in the pathogenesis of pseudomembranous colitis. Patients usually present with watery diarrhea, tenesmus, fever, and abdominal pain. Symptoms typically occur 7-10 days after initiation of the antibiotic; in rare cases, pseudomembranous colitis can occur up to 6 weeks after antibiotic initiation. Toxic megacolon, resulting from progressive loss of muscular tone in the colon, is a rare but serious complication that could progress to perforation of the colon and peritonitis. On sigmoidoscopy or colonoscopy, the disease is characterized by an accumulation of an inflammatory exudate, the "pseudomembrane" over the mucosa. Assay for C. difficile toxins (enterotoxin A or cytotoxin B) is useful in diagnosing pseudomembranous colitis. . Oral metronidazole is the treatment of choice but another option is oral vancomycin.

A 40-year-old obese white woman, mother of 5 children, gives a history of repeated episodes of right upper quadrant abdominal pain. The pain is triggered with the ingestion of fatty foods and relieved with anticholinergic medications. The pain is spasmodic, radiates to the right shoulder and around to the back, and is accompanied by nausea and occasional vomiting. The patient is in no pain at the moment but is anxious to avoid further episodes. She is afebrile, and physical examination is unremarkable. Which of the following is the most appropriate next step in management?

The clinical description is classic for biliary colic caused by gallstones that are intermittently impacted at the cystic duct. The diagnostic study of choice to confirm the presence of gallstones is a sonogram.

A 56-year-old alcoholic man gives a history of several years of constant epigastric pain, radiating straight through to the back. The pain is severe, present at all times, and exacerbated by eating. He also has steatorrhea and diabetes. He relates that he began to have episodes of acute alcoholic pancreatitis in his mid-thirties. At first these occurred every few years, but eventually they grew in number to several attacks per year. Eventually the pain became constant. Although he made numerous attempts to quit drinking, he was not successful until about 1 year ago; still, his current abstinence has not alleviated the pain. Which of the following is the most appropriate initial step in evaluation?

The clinical diagnosis is chronic pancreatitis, and CT scan will give useful information about potentially correctable features of the disease, such as pseudocysts, dilated ducts, calculi, or areas amenable to resection. CT scan of the abdomen is considered to be the most sensitive test for picking up chronic pancreatitis. If surgery is contemplated, ERCP will then be needed.

A 42-year-old alcoholic man comes to the emergency department with severe abdominal pain that began 3 hours ago. He is well known to the medical staff because of 3 prior hospitalizations for acute pancreatitis. He says that this time his pain began suddenly and in short order became constant, generalized, and extremely severe. He lies motionless on the stretcher, and on physical examination he has tenderness, rigidity, and rebound in all four abdominal quadrants. Bowel sounds are absent. Initial studies are significant for a serum amylase of 1,200 U/L and free air under both diaphragms on an upright plain film of the abdomen. Which of the following is the most likely diagnosis?

The clinical picture is highly suggestive and the radiograph findings virtually pathognomonic for a perforated viscus. Alcoholics can easily get peptic ulcer disease (which can perforate), and perforated ulcers are well known for their propensity to produce elevations of the serum amylase. Patients who have a perforated viscus will classically lie motionless and have a rigid, quiet abdomen on physical examination. The next step in any patient with a perforated viscus is emergency laparotomy.

A 21-year-old college senior comes to the health care provider with a 2-month history of frequent episodes of loose stool, preceded by lower abdominal cramping. Over the past 4 weeks, the stools have become increasingly bloody. On a number of occasions he has had the sensation of rectal fullness but is unable to pass any fecal matter. A sigmoidoscopy reveals inflammation in a circumferential pattern from the anal verge to the mid-sigmoid colon, where a transition to normal mucosa is seen. Which of the following is the most appropriate treatment for this patient?

The patient described here has ulcerative colitis confined to the distal colon, also known as ulcerative proctosigmoiditis. Because the disease is limited to the distal colon, topical agents such as mesalamine (or alternatively hydrocortisone) would be effective in reducing inflammation. Mesalamine is an anti-inflammatory drug used principally to control ulcerative colitis. Its active ingredient is also known as 5-aminosalicylic acid, which is available in the forms of rectal suspension, suppositories, delayed release oral tablets, and controlled release oral capsules. The mode of action is unknown but is thought to involve topical (because mesalamine is poorly absorbed), rather than systemic, modulation of arachidonic acid metabolites, including prostaglandins, leukotrienes, and hydroxyeicosatetraenoic acids. It is usually well tolerated, but it can cause significant allergic reactions related to sulfite sensitivity. Ulcerative colitis treatment is dependent on the activity of the disease and the extent of colon involvement. The goal of therapy is to induce and to maintain remission. For mild to moderate disease affecting the distal colon, topical mesalamine is clearly a first-line choice. This is also the treatment of choice for distal colitis that is in remission. Oral mesalamine may be needed for patients who have extensive disease of the colon that has extensive colitis.

A 31-year-old woman smashes her car against a bridge abutment. She sustains multiple injuries, including upper and lower extremity fractures. She is fully awake and alert, and she reports that she was not wearing a seat belt and distinctly remembers hitting her abdomen against the steering wheel. Her blood pressure is 135/75 mm Hg and pulse88/min. Physical examination reveals a rigid and tender abdomen. There is severe tenderness when external pressure is applied to her abdomen and then suddenly released. She has no bowel sounds. Which of the following would be the most appropriate step in evaluating potential intra-abdominal injuries?

The presence of an "acute abdomen," which this woman has, is an indication for exploratory surgery and prompt repair of the injuries (probably affecting hollow viscera) that have produced the signs of peritoneal irritation.

A 70-year-old man comes to the emergency department complaining of abdominal pain. He describes the pain as crampy and primarily located in his left lower quadrant. He has had minimal nausea, but complains of constipation. His past medical history is significant for hypertension, hyperlipidemia, and gout. His medications include atenolol and simvastatin. He is allergic to penicillin. His temperature is 38.0º C (100.4º F), blood pressure is 140/60 mm Hg, pulse is 100/min, and respirations are 20/min. His physical examination is significant for tenderness to palpation at the left lower quadrant without rebound or guarding. His stool is negative for occult blood. His heart and lung examinations are unremarkable. Which of the following is the most likely diagnosis?

The presence of cramping, left lower quadrant pain associated with fever and constipation is a classic presentation of diverticulitis. Acute diverticulitis consists of left lower quadrant pain, tenderness, fever, and constipation. Rebound tenderness in the left lower quadrant may occur, and CT scan can show thickening of the bowel wall, mass, abscess, and some streakiness in the mesenteric fat. Treatment is with antibiotics and analgesia.

A 53-year-old man is admitted to the hospital for fever and abdominal pain. He has a history of cirrhosis and is known to be hepatitis C positive. He was diagnosed with cirrhosis 4 years ago. He denies any alcohol or tobacco use. His only medications are spironolactone (Alactone) and propranolol (Inderal). He reports that 5 days ago he had a temperature of 38.9º C (102º F) along with a gradual onset of diffuse abdominal pain. On examination his temperature is 38.3º C (101º F), blood pressure 100/50 mm Hg, and pulse 110/min and regular. Lungs are clear. There are numerous spider angiomata on the thorax and back, and the abdomen appears distended.

The prevalence of infection of ascites fluid, so called spontaneous bacterial peritonitis (SBP), is estimated at 50% in hospitalized cirrhotic patients. This patient has had long-standing cirrhosis, has an extremely low serum albumin level, and is on spironolactone (a direct aldosterone antagonist), which is prescribed to patients with ascites. All febrile patients admitted with ascites must have abdominal paracentesis performed to both determine the cause for the ascites and rule out infection of the ascites. A diagnosis of SBP is made when there is an elevated ascitic fluid absolute neutrophil count (>250 cells/mm3) without an evident intra-abdominal or surgically treatable cause for the infection. SBP most often occurs with portal hypertensive ascites. The most sensitive marker available for such ascites is a serum/ascites albumin gradient >1.1 g/dL. Abdominal CT scan is only indicated after an abdominal paracentesis is performed and SBP is excluded. All febrile patients admitted with presumed ascites must undergo abdominal paracentesis to determine the cause of the ascites and rule out infection.

A 79-year-old man with atrial fibrillation develops an acute abdomen. When seen 2 days after the onset of the abdominal pain, he has a silent abdomen, with diffuse tenderness and mild rebound. There is a trace of blood on the rectal examination. He also has acidosis and looks quite sick. Radiographs show distended small bowel and distended right colon, up to the middle of the transverse colon. Which of the following is the most likely diagnosis? Acute pancreatitis, mesenteric ischemia, midgut volvulus, perforated viscus, primary peritonitis

The setting of an elderly patient who has atrial fibrillation (or a recent myocardial infarction) who develops an acute abdomen strongly suggests embolus to the mesenteric vessels. The combination of abdominal pain and a trace of blood in the lumen is also typical, as is the radiograph outlining the territory supplied by the superior mesenteric artery. The patient probably has a dead bowel by now, as evidenced by his acidosis and severe illness. Any one of the other options could exist, but none are the most likely. Embolic phenomena to the superior mesenteric artery may occur in the setting of atrial fibrillation. Cardiovascular disease with atherosclerosis is also a major risk factor for ischemic bowel disease since blood flow to the bowel may not be sufficient, causing chronic ischemic symptoms often manifested by abdominal angina. The underlying cause of the ischemic bowel must be corrected (embolectomy or revascularization) to save the bowel. Gangrene of the bowel secondary to ischemia has a high rate of mortality.

A 25-year-old man sustains multiple stab wounds to the abdomen when he is mugged while jogging. The assault takes place in the evening. He is dumped by the attackers behind thick vegetation and is not found until the next morning, at which time exploratory surgery reveals multiple small bowel and colonic lacerations. All of the lacerations are repaired. In the post-op period, the patient has persistent hypotension, even though he received adequate fluid infusion and his central venous pressure is 12 mm Hg. Further studies done with the help of a pulmonary artery catheter reveal high cardiac output and low peripheral resistance. Which of the following is the most likely diagnosis?

The signature of septic shock in the normovolemic patient is high cardiac output and low peripheral resistance. The findings are classic, and the circumstances (massive peritoneal bacterial contamination) easily explain its development. Septic shock: high cardiac output and low peripheral resistance leading to hypotension. Hypovolemic shock: low CVP and low cardiac output. Neurogenic shock: low CVP and low cardiac output. Cardiogenic shock: high CVP and low cardiac output caused by pump failure activity of the heart.

A 9-month-old girl is seen in the health care provider's office because of failure to gain weight. Her length and weight are both below the fifth percentile for her age. The patient's chart indicates that at age 6 months, her length and weight were at the fiftieth percentile. A careful history reveals that the mother returned to work when the infant was 6 months old, and the grandmother has assumed the care of the infant since then. She receives 6-8 oz of iron-fortified, cow protein-based formula every 4 hours. Which of the following is the best initial step in the management of this infant?

This 9-month-old infant presents with failure to thrive, which is determined by the decline in her length and weight to below the fifth percentile. Failure to thrive can be either organic or inorganic (i.e., social). Although it is important to identify the causes of organic failure to thrive, most cases are inorganic. In this clinical vignette, because the reduction of weight happened after the mother returned to work, an inorganic cause is likely. One of the most common causes of failure to thrive is improper preparation of the formula, resulting either from an incorrect water-to-formula ratio or from poor mixing techniques. In addition to obtaining a detailed history of how the formula is mixed, it might be very useful to have the caretaker actually demonstrate how he or she prepares the formula. In this case, the most likely reason that the infant has failure to thrive is that the grandmother has improperly mixed the formula. Also, a 9-month-old infant should be eating some regular table foods, including meats, poultry, and vegetables.

A 44-year-old obese woman comes to the emergency department complaining of 3 hours of severe abdominal pain. She has also had multiple episodes of vomiting during this time. She describes the pain as "worse than labor," and it radiates to the interscapular region. Her temperature is 38.9ºC (102ºF), and there is severe tenderness in the right upper quadrant. She reports that she has had multiple similar episodes in the past which have lasted approximately 30 minutes and then resolved spontaneously. Which of the following ducts is most likely being obstructed by a gallstone?

This patient has acute cholecystitis. Risk factors include female gender, obesity, and a classic history of prolonged biliary colic in association with fevers. The presentation illustrated here is typical and results from obstruction of the cystic duct, which drains the gallbladder. Obstruction of the common bile duct or the pancreatic duct will produce acute ascending bacterial cholangitis, which would be demonstrated by Charcot triad (i.e., right upper quadrant pain, fever, and jaundice). Obstruction of either the common hepatic duct or the right hepatic duct may give a limited episode of cholangitis but will not cause cholecystitis, because the obstruction occurs in the biliary tree above the level of the entry of the cystic duct.

A 5-year-old boy is brought to the emergency department with 2-day history of fever, anorexia, loose stools, and yellow skin color. He attends a large daycare center. On physical examination his temperature is 38.1ºC (100.7ºF), blood pressure 88/56 mm Hg, pulse 74/min, and respirations 15/min. Initial laboratory evaluation reveals a total bilirubin of 1.8 mg/dL (normal <1.9 mg/dL) and alanine aminotransferase 764 U/L (normal <25 U/L in this age group). Nobody else in his family is sick. Which of the following is the most appropriate diagnostic test?

This boy most likely has hepatitis A. While it is more common in developing countries, it is still a common infection in the developed world. It is transmitted by the fecal-oral route when contaminated food or water is ingested. The incubation period is about 15-40 days. Ninety percent of children acutely infected with hepatitis A are asymptomatic. The virus is shed in the feces 2-3 weeks before the onset of jaundice until approximately 1 week after onset. Most infected children are infectious for a long time before they are symptomatic. Hepatitis A is therefore very difficult to control. Large outbreaks frequently occur in daycare centers. Diagnosis is best made by determination of IgM levels against hepatitis A virus. The presence of IgM for hepatitis A suggests acute infection; this antibody peaks at 4-6 weeks and does not persist beyond 6 months. IgG is produced in the primary infection, but for most viral infections, including hepatitis A, it persists for a lifetime.

An 82-year-old woman is brought to the emergency department from the local nursing home because of complaints of severe constipation. She has not moved her bowels in 5 days and today began complaining of lower abdominal discomfort and distention. She takes hydrochlorothiazide and acetaminophen with codeine for severe arthritic pain in both hips. Over the past week she has been bedridden because of the severe pain. She is afebrile. Abdominal examination shows mid- and lower-abdominal distention with mild tenderness. Bowel sounds are normoactive. Rectal examination demonstrates that the rectal vault is filled with hard stool. Which of the following is the most appropriate next step in management?

This elderly woman, who has been bedridden and on a narcotic analgesic, has become severely obstipated. This multifactorial condition is caused by her immobilization, bowel hypomotility secondary to the codeine, and possible mild dehydration from her diuretic. Treatment is manual disimpaction, which will promptly "open the flood gates" and yield immediate relief for the patient.

A 12-month-old boy is brought to the emergency department with bilious vomiting and abdominal distention for 10 hours. His mother states that the infant has been constipated since birth and failed to pass meconium during the first 48 hours of life. When specifically asked, she says that he often has brown, speckled vomitus. On examination he is very irritable. His length and weight are both below the fifth percentile for his age. The abdomen is moderately distended. After a digital rectal examination, a fair amount of stool ejects out from the anus. There is no stool in the ampulla. Which of the following is the most likely diagnosis?

This infant has Hirschsprung's disease, or congenital aganglionic bowel disease. It is 5 times more common in boys than in girls. It results from congenital absence of ganglion cells in either part of or the entire wall of the colon, resulting in a state of tonic contraction. In most cases, the aganglionic segment is limited to the rectosigmoid colon. In very rare cases, part of or the entire small bowel can be aganglionic as well. Bilious or feculent vomiting, abdominal distention, and constipation are the classic clinical signs. There might also be a history of failure to pass meconium in the first 48 hours of life. If only a short segment of the colon is involved, Hirschsprung's disease might not be evident until childhood or adolescence. Megacolon proximal to the aganglionic segment might be visible on barium enema. The diagnosis is confirmed with the demonstration of an aganglionic segment of the bowel on punch biopsy.

A 68-year-old woman reports that she had an operation 40 years earlier for intractable ulcer symptoms. She recalls that a large portion of her stomach was removed. Review of her prior medical records indicates that she underwent an antrectomy and a gastrojejunostomy. Which of the following complications is most likely in this scenario?

This patient had a Billroth II gastrojejunostomy, which is a commonly used operation for both intractable peptic ulcer disease and gastric carcinoma. In this operation, the distal approximately two thirds of the stomach is resected, and the stomach is then attached along a longitudinal plane to the jejunum. The technique usually leaves a foot or more of duodenum in a "blind loop" configuration through which food does not pass. The operation does, however, allow preservation of the extrahepatic duct system, so that biliary and pancreatic secretions can still enter the gut. Patients are at risk for bacterial overgrowth in the blind loop. Blind loop syndrome can be a cause of steatorrhea related to bacterial destruction of bile salts, macrocytic anemia related to malabsorption of vitamin B12, and diarrhea and malnutrition related to generalized malabsorption of nutrients secondary to bacterial destruction of pancreatic enzymes. This blind loop may allow the retained bacteria to make more vitamin K, which is important to remember in the setting of a patient taking warfarin (Coumadin).

A 19-year-old college freshman undergoes sigmoidoscopy because of a family history of multiple polyps in his young siblings. His brother underwent total proctocolectomy at age 23, and his sister underwent a total proctocolectomy at age 29, after both were found to have hundreds of colonic adenomas on colonoscopy. Both siblings are alive and well 5 years later and without any other findings of neoplasms. The patient undergoes sigmoidoscopy and is found to have several dozen small colonic polyps within the rectosigmoid. Which of the following is the most appropriate next step in management?

This patient has 2 first-degree relatives who have undergone total proctocolectomies for a polyposis syndrome. This most likely represents the familial polyposis coli syndrome in that both siblings had high numbers of colonic adenomas and underwent an appropriate total proctocolectomy. If this patient is found to have several dozen polyps during the flexible sigmoidoscopy, it is evidence that he is expressing the phenotypic trait of the familial adenomatous polyposis (FAP) gene. Rather than wait for any of these polyps to undergo malignant degeneration, he should have a total proctocolectomy in the immediate near future. Once many polyps have been demonstrated, the likelihood of one of these becoming malignant is high.

An 8-year-old girl is brought to the office because she has been complaining of frequent abdominal pain. The episodes occur every several days, show no particular pattern, and resolve without treatment within 10-15 minutes. During the episodes, the child has to lie down and bring her legs up to her chest for relief. The parents state that she started having this problem approximately a year ago, and they believed it was related to her diet. After trying to restrict various kinds of food with no effect, they decided to have her examined. Her bowel movements are regular. She has no significant past medical history and takes no medication on a regular basis. She has an older brother who is in excellent health. The mother tells you that she loves school and is a very good student. She is very popular among her friends and has no problems in social relationships. Of interest, they immigrated to the United States 3 years earlier and have moved 3 times since then for work. She did not seem to have any trouble adjusting to that situation. They will be moving to another city again in 6 months. On physical examination the girl is in no acute distress. She is in the 90th percentile for height and weight. Vital signs are within normal limits, as is a complete blood count with white blood cell differential. Her abdomen is not distended and is soft and nontender to palpation. Plain abdominal films show no abnormalities. Which of the following patient intervention/patient education scenarios should be pursued?

This patient has functional recurrent abdominal pain and it is >highly likely that she will continue to have episodes of abdominal pain even as an adult.< Patients who have functional recurrent abdominal pain present with nonspecific symptoms. It is difficult to obtain a pattern of occurrence of the pain. Stressors such as school or examinations may produce a predictable pattern. Social factors, such as relocating, family illness, and sibling rivalry may account for abdominal pain. Medically unexplained headache or abdominal pain occurs at least once a week in 10 to 30% of children and adolescents in the United States. Seventy percent of children will continue to experience abdominal pain into adulthood, although the symptoms may not impair their physical activity to the same degree. Up to 30% will develop additional complaints, such as headaches. Parents may have suffered from abdominal pain. Treatment may include counseling, relaxation training, behavioral methods, biofeedback, and family therapy, together with treatment of possible comorbid conditions, such as anxiety or depression. After a thorough history and physical examination, most patients who have recurrent abdominal pain do not require further evaluation. Useful studies, however, include complete blood count, sedimentation rate, urine analysis, stool studies, ultrasound, radiographic studies, breath hydrogen, and endoscopy.

A 35-year-old clergyman with 19-year history of type 1 diabetes is referred to a gastroenterologist by his endocrinologist because of symptoms of early satiety, abdominal bloating, nausea, and recurrent episodes of vomiting after a large meal. A physical examination reveals mild retinopathy and 2 healed ulcers on the ventral surface of his left foot. Abdominal examination is normal. Which of the following is the most likely cause of his symptoms?

This patient has gastroparesis as a result of autonomic neuropathy caused by long-standing diabetes. Autonomic neuropathy is usually seen in diabetic patients who have polyneuropathy, but it can also be associated with postural hypotension, disordered sweating, impotence in men, bladder dysfunction, esophageal dysfunction, and constipation or diarrhea. As with other forms of diabetic peripheral neuropathy, it has been speculated that damage to the tiny blood vessels that accompany and feed nerves may contribute to the nerve dysfunction. If autonomic dysfunction is suspected in a diabetic patient, a useful maneuver on physical examination is to look for a decrease in heart rate in response to the Valsalva maneuver (forced expiratory effort against a closed airway). Autonomic neuropathy is also detected with the lack of pulse variation in naturally occurring cardiac sinus arrhythmia. Diabetic gastroparesis is difficult to manage; patients who cannot tolerate meals of ordinary size and type may tolerate many tiny meals with the use of fluid nutritional supplements to ensure adequate nutrition. These patients may also have retained food in their stomach hours after a meal because of the gastroparesis, which can lead to complications in the setting of emergency surgery with intubation.

A 51-year-old welder comes to the health care provider complaining of severe fatigue and the onset of jaundice. He has a known history of hepatitis C, which he acquired after intravenous drug use 20 years earlier. Over the past 6 months he has developed ascites and has had two admissions to the hospital for esophageal variceal bleeding. On physical examination he is icteric with bitemporal wasting and multiple stigmata of chronic liver disease. On abdominal examination, the liver is percussed to be 7 cm in the midclavicular line and splenomegaly is present. There is near-tense ascites and moderately severe lower extremity edema that extends to the mid-calf. Laboratory results reveal an albumin of 2.1 g/dL (normal 3.4-5.4 g/dL), total bilirubin 12.1 mg/dL (normal <1.9 mg/dL), and prothrombin time 19 seconds (normal 11-13 seconds). Which of the following is the most appropriate therapy?

This patient has known hepatitis C. Features indicating that he has advanced signs of cirrhosis and portal hypertension include ascites, splenomegaly, jaundice with elevated bilirubin, hypoalbuminemia with leg edema, increased prothrombin time probably secondary to inadequate synthesis of clotting factors by the liver, and esophageal variceal bleeding. In the setting of disease this advanced, antiviral treatment for hepatitis C is without value and the patient should undergo evaluation for a liver transplant.

A 61-year-old man is found to have a malignant polyp in the cecum. He undergoes a right hemicolectomy, and the specimen reveals a 2-cm adenocarcinoma extending into, but not through, the muscularis propria. Eleven lymph nodes are negative and there is no evidence of distant metastatic spread. He recovers uneventfully and is discharged home on the postoperative day 6. Four weeks later he develops sudden onset of abdominal distention with vomiting and the inability to pass flatus. Rectal examination reveals no masses and brown stool that is negative for occult blood. An abdominal radiograph is pending. Which of the following is the most appropriate next step in management?

This patient has signs and symptoms of a small bowel obstruction. Given the histologic findings of the surgical specimen of an early (Duke B1) lesion, it is highly unlikely that there is recurrent tumor, especially given the prompt occurrence of this bowel obstruction. The most likely etiology is therefore adhesions, which have caused this obstruction. Placement of a nasogastric tube would be the most appropriate measure to decompress the bowel. In most of these patients, this approach will lead to gradual and complete resolution of the bowel obstruction. Should there be any symptoms of clinical deterioration with the placement of a nasogastric or small intestinal tube, then surgery should be considered. In the setting of a bowel obstruction, IV fluids, a nasogastric tube, and bowel decompression are indicated. Most patients will improve with conservative management; if the patient fails to do so, then surgery must be considered.

A 45-year-old woman has been experiencing generalized weakness and a sensation of "pins and needles" for the past 3 week. She exercises daily, rarely drinks alcohol, and is a strict vegetarian. Her temperature is 37ºC (98.6ºF), blood pressure 110/70 mm Hg, pulse 60/min, and respirations 18/min. Examination shows weakness of the proximal and distal muscles of the lower extremities. Deep tendon reflexes are increased. The gait is ataxic. Which of the following is the most likely cause of these symptoms?

This patient has subacute combined degeneration of the spinal cord, which is caused by a vitamin B12 deficiency. It is most often caused by pernicious anemia, but it may be acquired by patients who have strict vegetarian diets or small bowel disease. The clinical manifestations include weakness, paresthesias, loss of vibratory sensation, increased deep tendon reflexes, and extensor plantar responses. The gait is ataxic. Mental changes may also occur. The diagnosis is made by measuring serum vitamin B12 levels. Vitamin B12 deficiency occurs only after years of being a vegetarian, because vitamin B12 stores are present for years before depletion occurs. Neurologic changes caused by pernicious anemia are considered to be irreversible, so early treatment is warranted. The treatment is vitamin B12 replacement.

A 43-year-old man reports a 9-pound weight loss over the past 9 months, accompanied by difficulty swallowing solids and liquids. He has woken on several occasions at 4 AM and regurgitated partially digested dinner contents. An upper gastrointestinal series reveals a widely dilated esophagus with a smoothly tapering distal esophagus. There appears to be partially digested food present in the esophagus. Which of the following is the most likely diagnosis?

This patient has the symptoms of a motility-type dysphagia in that he has difficulty with both solids and liquids from the onset of his symptoms. The nocturnal aspiration of food occurs because the esophagus remains filled for hours or even days after completing a meal. The radiograph reveals the typical dilated esophagus of achalasia, which is termed a bird's-beak esophagus, with distal esophageal tapering. Achalasia is a motor disorder affecting the esophagus. There are two simultaneous problems that occur with this disorder: failure of the lower esophageal sphincter to relax and poor peristalsis in the distal third of the esophagus. The end result of this condition is that food is retained in the lower esophagus with dilation of the distal esophagus and poor transfer of food into the stomach. Patients present with dysphagia for solids and liquids along with regurgitation and retrosternal chest pain. Definitive therapy is surgery with pneumatic dilation versus laparoscopic cardiomyotomy. Conservative therapy may also include nitrates or calcium-channel blockers that help to relax the lower esophageal sphincter, which helps to empty the food from the esophagus.

A 4-month-old girl is brought to the office because of frequent crying, sleep disturbance, and decreased appetite. She has been irritable and crying a great deal since birth, especially after feeding when she might spit up and even vomit. She also seems to get hiccups frequently. She has not been gaining weight as expected and her parents are concerned that something may be wrong. Last night they thought she had stopped breathing for approximately 20 seconds. She has been on a diet of cow's milk and breast milk. On physical examination the infant seems in no acute distress. Her vital signs are within normal limits and her weight and length are in the tenth percentile. Otherwise the physical examination is within normal limits. Which of the following is the most appropriate next step in management?

This patient has the typical presentation of gastroesophageal reflux. Gastroesophageal reflux occurs with a reduction in lower esophageal sphincter pressure, inappropriate lower esophageal sphincter relaxation, hiatal hernia, or delayed gastric emptying. This condition is common in the pediatric population, particularly in infants with developmental delay or cerebral palsy. It is a minor condition and of no consequence (so-called "functional gastroesophageal reflux"); patients present with a wide array of symptoms. Some form of spitting up and even forceful vomiting are common. Apnea can be a presenting sign. Reflux into the hypopharynx triggers laryngospasm and subsequent obstructive apnea. Chronic cough and wheezing may signal aspiration. Some patients exhibit poor weight gain and failure to thrive. Sandifer syndrome presents with gastroesophageal reflux and opistotonus, presumably to avoid aspiration or decrease pain. A pH probe is the standard method that is used for diagnosing gastroesophageal reflux. Other imaging studies include technetium scanning and barium swallow. Initial therapy consists of antireflux measures, such as elevating the head of the bed and thickening of feeds. If this fails to provide relief of symptoms, medical management with antacids, prokinetics, H2-receptor blockers, and proton pump inhibitors should be attempted. Failure of medical management may require surgical correction with a Nissen fundoplication. Most patients, however, have resolution of symptoms without any treatment.

A 24-year-old dress designer complains of a crampy periumbilical pain over the past 9 months since she began her first job. During that time she has experienced several episodes of constipation lasting 4-5 days, typically followed by 3-4 days of frequent loose bowel movements. She denies any bloody stools, fever, weight loss, or change in appetite. Her symptoms are generally milder on weekends. Physical examination is normal. White blood cell count is 6,700/mm3 (normal 5,000-10,000/mm3), hematocrit 38%, (normal 36-44%), and erythrocyte sedimentation rate 4 mm/h (normal <25 mm/h in women). Serum albumin and liver function tests are normal. Which of the following is the most likely diagnosis?

This patient has typical crampy abdominal symptoms with alternating diarrhea and constipation. She is a young individual who has chronic symptoms, which is very typical of irritable bowel syndrome. Patients who have this diagnosis will generally have symptoms of pain related to meals or stress and an alteration in bowel habits. This change in bowel pattern may take the form of frequent loose stools versus constipation, or a combination of both. These patients have no physical findings or laboratory results to suggest an inflammatory process. Dietary changes and stress reduction are usually the goals of therapy, with antispasmodic drug therapy for symptomatic relief.

An 18-year-old college student without a significant past medical history comes to the health care provider complaining of a 2-week history of bloody diarrhea, vague intermittent abdominal pain, and a 15-pound weight loss. He denies any sick contacts, travel, or eating any foods that were "new or unusual." His temperature is 38.3ºC (100.9ºF), blood pressure 100/70 mm Hg, pulse 93/min, and respirations 13/min. Physical examination reveals a thin young man in no distress. His abdomen is mildly diffusely tender without rebound or guarding. Rectal examination is hemoccult positive. Laboratory studies show a mild iron-deficiency anemia and a mildly elevated sedimentation rate. Colonoscopy reveals continuous and extensive inflammation of the colon without skip lesions. Which of the following is associated with this patient's condition?

This patient has ulcerative colitis. This question highlights the differences between ulcerative colitis and Crohn's disease. The presence of continuous inflammation as opposed to "skip lesions," bloody diarrhea, and rectal involvement all suggest ulcerative colitis. Colectomy is curative but is usually reserved for treatment-resistant cases or patients who have active ulcerative colitis for longer than 10 years, because those patients are at high risk for colon cancer. Ulcerative colitis is characterized by diffuse inflammation of the colonic mucosa, which begins in the rectum and progresses in a stepwise fashion proximally. It is characterized by a relapsing, remitting course. Treatment is intended to induce and maintain remission. Complications of ulcerative colitis include toxic megacolon (risk for rupture) and adenocarcinoma of the colon, which is seen in 3 to 5% of the patients.

A 36-year-old woman without significant past medical history is complaining of 8 weeks of intermittent diarrhea. She reports the diarrhea is nonbloody and without mucus. There is mild abdominal cramping but no severe pain. There is no associated constipation, and symptoms are not worse during stressful periods. The diarrhea occurs shortly after meals and improves with fasting. She denies any weight loss. She denies sick contacts, travel history, camping history, or eating at any unusual locations. She is unsure if there is an association with milk products. Her temperature is 36.7ºC (98ºF), blood pressure is 100/70 mm Hg, pulse is 73/min, and respirations are 13/min. Her abdomen is soft, nontender, and nondistended with normal bowel sounds. Which of the following is the most likely etiology of her diarrhea?

This patient is describing lactose intolerance. There are several clues to the diagnosis and reasons the other choices are incorrect. Lactose intolerance is very common. It is an osmotic diarrhea, which means that undigested food causes the symptoms. Resolution with fasting is another sign that the diarrhea is osmotic. The only answer choice listed that is osmotic is lactose intolerance. In addition, the lack of red flag symptoms such as significant abdominal pain, weight changes, or bloody stools leads us to believe that this is not a more serious condition. Because ingestion of lactose-containing food is the precipitating cause, fasting will improve these symptoms.

A newborn examination of an infant reveals a scaphoid abdomen and a palpable fullness of the epigastrium. An abdominal radiograph of the infant shows gaseous distention of the stomach and proximal duodenum. A nasogastric tube is placed and suction produces bilious fluid from the stomach. Which of the following prenatal studies might have revealed this abnormality? AFP, Antibody screen, Hb electrophoresis, hCG levels, ultrasound

This patient most likely has duodenal atresia, an obstruction resulting from a failure of recanalization of the duodenal lumen, resulting in complete intrinsic obstruction. It can be detected with prenatal ultrasonography and it occurs in 1/10,000 live births. Twenty to 30% of patients with duodenal atresia have trisomy 21. There is no predilection for sex or race. Duodenal atresia presents early, usually in the first hours or day of life. The neonate has bilious vomiting without abdominal distention. In approximately 15% of neonates who have this anomaly, vomiting is nonbilious because the obstruction is located proximal to the ampulla of Vater. There may be a history of polyhydramnios (which is commonly associated with diabetes and twin pregnancy). Prenatal ultrasonography may reveal a dilated and fluid-filled stomach and duodenum in addition to other possible anomalies. On physical examination, the newborn may have a scaphoid abdomen and epigastric fullness from a dilated stomach and proximal duodenum. Nasogastric suction produces bilious fluid. Abdominal films reveal the characteristic "double bubble" of the stomach and proximal duodenum. Therapy involves surgical correction. The patient should be evaluated for associated anomalies.

A 72-year-old woman comes to the emergency department complaining of severe left lower quadrant pain for 24 hours. She has a long history of constipation and frequently has left lower quadrant cramping after meals. Over the past 24 hours, she has had increasing discomfort in the left lower quadrant and has had a temperature to 39.4ºC (102.8ºF). On examination there is tenderness to palpation and guarding in the left lower quadrant. CT scan reveals an abscess involving and contiguous to the sigmoid colon. Which of the following is most likely causing this patient's condition?

This question illustrates the classic presentation of acute diverticulitis. It is important to distinguish diverticulosis, the presence of diverticula in the colon, and acute diverticulitis, which implies acute inflammation of a diverticulum. Diverticulosis is primarily a disease of the elderly and of individuals who consume a low fiber diet and it results in the development of hundreds of diverticula. Most of the diverticula tend to develop in the very distal colon (where the stool tends to be the most dehydrated and hardest to move by peristalsis), although in severe cases even the ascending colon may be involved. This predilection for distal colonic involvement tends to clinically produce chronic constipation and left lower cramping after meals, as seen in this patient. Acute diverticulitis should be suspected in cases like this one. Most cases of acute diverticulitis occur as the result of a micro- or macroperforation of a diverticulum, which allows gut bacteria to escape from the bowel lumen. In this case, a transmural perforation has created an abscess in the region of the sigmoid colon.

A 53-year-old woman comes to the health care provider complaining of fatigue over the past 6 months. During this time, she has also developed pruritus and lost 4 pounds. She is not sexually active and her past medical history is significant only for Sjögren syndrome. On physical examination she is afebrile and has mildly icteric sclera. There are excoriations noted on all 4 extremities and trunk and back. The liver edge is smooth and non-tender and measures 9 cm at the midclavicular line. There is no ascites, splenomegaly, or peripheral edema. Laboratory results reveal a normal complete blood count, normal electrolytes, liver function tests revealing an alkaline phosphatase of 260 U/L (normal <110 U/L), total bilirubin of 3.1 mg/dL (normal <1.9 mg/dL), and normal transaminase levels. Which of the following is the most likely diagnosis?

This woman has a classic presentation of primary biliary cirrhosis. It typically affects middle-aged women and will progress gradually to the point of end-stage liver disease over a number of years. The disease is caused by an autoimmune destruction of intrahepatic bile ducts, and the diagnosis is made by liver biopsy. The serology that should be checked is the antimitochondrial antibody. Primary biliary cirrhosis is often seen in individuals who have other autoimmune diseases, such as Sjögren syndrome, pernicious anemia, and Hashimoto thyroiditis. Primary biliary cirrhosis most commonly occurs in women over age 45. It is a progressive disease that classically presents with pruritus and fatigue. The pruritus occurs as a result of a toxic accumulation of bile salts retained in the liver, which also leads to progressive liver damage and cirrhosis. Autoantibodies in the form of antimitochondrial antibodies are seen in this autoimmune disease. Treatment is bile salt therapy (ursodiol or ursodeoxycolic acid), which can slow or stop the progression to advanced liver disease, and cholestyramine which controls the itching. End-stage liver disease can be treated only with liver transplantation.

A 42-year-old obese woman experiences episodic abdominal pain. She notes that the pain increases after the ingestion of a fatty meal. Action of which of the following hormones is responsible for the postprandial intensification of her symptoms?

This woman has a risk profile (female, fat, forties) and symptomatology consistent with gallstones (cholelithiasis). As would be expected, contraction of the gallbladder following a fatty meal often exacerbates the pain caused by gallstones. Cholecystokinin (CCK) is the hormone responsible for stimulation of gallbladder contraction; the release of CCK is stimulated by dietary fat. It is produced in I cells of the duodenum and jejunum. In addition to gallbladder contraction, CCK also stimulates pancreatic enzyme secretion and decreases the rate of gastric emptying.

A 36-year-old man with ulcerative colitis develops pruritus and fatigue. Alkaline phosphatase is elevated. The biliary tree appears beaded on barium radiograph. Which of the following is the most likely diagnosis?

Young men who have ulcerative colitis are at increased risk for developing primary sclerosing cholangitis (PSC), a chronic cholestatic condition that leads to fibrosis of the bile ducts. This cholestasis leads to an elevation in the alkaline phosphatase level. A classic clue to the diagnosis is a beaded appearance of the biliary tree on barium radiograph. Sclerosing cholangitis is a condition in which there is scarring in the bile ducts, leading to cholestatic liver disease, cirrhosis, and end-stage liver disease. It is most common in young and middle-aged men, especially those with underlying inflammatory bowel disease. Pruritus and jaundice are presenting symptoms. Serum alkaline phosphatase levels are elevated along with serum gamma-GT levels. When hepatocellular damage occurs, there is elevation of aminotransferase levels and conjugated bilirubin. ERCP and MRCP studies will show tight narrowing in the extrahepatic biliary tree caused by progressive structuring. Typically there is both intrahepatic and extrahepatic duct involvement.

A 24-year-old man comes to the clinic for evaluation of a 4-month history of postprandial diarrhea, weight loss of 9 pounds, and lower abdominal pain. He denies recent travel or antibiotic use. On physical examination his temperature is 38.0ºC (100.4ºF), and he has several oral aphthous ulcers. On abdominal examination there is tenderness and mild voluntary guarding in the right lower quadrant. Rectal examination reveals brown stool that is strongly positive for occult blood. Which of the following is most likely causing this patient's symptoms?

Transmural inflammation in the region of the terminal ileum. This patient, who has postprandial diarrhea, weight loss, low-grade fever, and right lower quadrant findings on physical examination, has the typical presentation of Crohn's disease, which most commonly involves the terminal ileum. Inflammation in this disease is transmural, as opposed to the inflammation in ulcerative colitis that is limited to the mucosa of the large intestine. Crohn's disease patients can present with chronic diarrhea, weight loss, and right lower quadrant abdominal pain. The diarrhea is caused by decreased absorption of bile acids with resultant secretory diarrhea. This depletion of the bile salt pool leads to malabsorption of fat with resultant steatorrhea and increased risk for gallstone formation. Fat soluble vitamins then become depleted. Diagnosis is confirmed with colonoscopy with biopsy. Because transmural inflammation of the bowel is the rule with Crohn's disease, bowel complications include obstruction, abscess formation, sinuses, and fistulae formation.

A 56-year-old man presents with complaints of gnawing pain in the mid-epigastrium, with occasional radiation to the back. He also notes a 15-pound weight loss over the past 3 months. The clinician suspects pancreatic carcinoma. Which of the following tumor markers would aid in confirming this diagnosis? AFP, CA-125, CA 19-9, hCG, PSA

Tumor markers can be very helpful in narrowing the possible primary sources for metastatic lesions. CA 19-9 is present in about 80% of patients who have pancreatic cancer. Tumor markers should not be used as the primary tool for cancer screening, but they have considerable utility in the confirmation of the diagnosis, as well as for monitoring recurrence or response to therapy. CA 19-9 is used preoperatively in pancreatic cancer as a predictor for invasive disease and postoperatively after tumor resection to assess that the tumor was completely eradicated and to monitor the patient postoperatively to assess for recurrence of the tumor. Because of its inadequate sensitivity and specificity, it is not used as a general screen for pancreatic cancer in asymptomatic patients.

On postoperative day 5 after elective left hemicolectomy for chronic diverticular disease, a patient is draining copious amounts of clear pink fluid from his midline laparotomy wound. A medical student removes the dressing, confirms that it is soaked, and sees a normal-appearing fresh wound with a row of skin staples in place. The student asks the patient to sit up so he can get out of bed and be helped to the treatment room for a more thorough examination. When the patient complies, the wound opens widely, and a handful of small bowel suddenly rushes out. Which of the following is the most appropriate management at this time?

Until the patient attempted to get out of bed, he had a wound dehiscence that could have been handled by taping the wound securely. Once the bowel came out, the problem became an evisceration. Immediate surgical repair is mandatory. While setting it up, the bowel must be protected from drying out, and the patient must be protected from significant heat loss. The key is application of warm and moist dressings.

A 48-year-old woman with a long history of hepatitis C is admitted for hematemesis. She is stabilized in the emergency department, and an upper endoscopy reveals bleeding esophageal varices. On examination, she is also noted to have a moderate amount of ascites. Which of the following is the most appropriate next step in the management of her bleeding?

When varices are seen, the immediate appropriate management is endoscopic banding of these varices, which will usually immediately lead to cessation of bleeding. Esophageal sclerotherapy is also effective but is associated with a higher instance of post-procedure complications, such as esophageal ulceration, stricture, and bacteremia. Esophageal varices develop as a direct consequence of portal hypertension. Portal hypertension can occur as a result of cirrhosis of the liver, which may be caused by chronic alcohol use or infection such as hepatitis B or C. Acute esophageal bleeding can be managed with resuscitation and endoscopic ligation. Options for therapy include octreotide (Sandostatin) and vasopressin. Intrahepatic portocaval shunts may also be used to lower portal pressures or can be used in the setting of failed endoscopic band ligation. Balloon tamponade may provide adjunctive therapy to the intrahepatic portocaval shunt to control the bleeding until the shunt is able to drop the portal pressures. TIPS (transjugular intrahepatic portosystemic shunt) is used to divert blood flow from the portal venous system to the systemic venous system. It may be used to decrease esophageal venous pressure, but it is not used acutely in the management of bleeding esophageal varices.


Conjuntos de estudio relacionados

Life in Medieval Towns Notes Activity

View Set

Chapter 5- Civil Rights [National Government}

View Set